You are on page 1of 72
blic_of the Philippines — . PROFESSIONAL REGULATION COMMISSION janila BOARD OF MEDICINE PHYSICIANS Licensure Examination Sunday, March 8, 2020 BIOCHEMISTRY INSTRUCTION: Select the correct answer for each of the following questions. Mark only one answer for each item by shading the box Sorresponding to the letter oF your choice on the answer sheet provid STRICTLY NO ERASURES ALLOWED. MULTIPLE CHOICE 4 ' Li As the level of visceral fat increases there is a concomitant loss of peripheral tissue responses to insulin. Adipose tissue responses to : these metabolic changes include a reduced secretion of which of the following adipokines? . A. Adipsin C. Adiponectin B. I1-6 D. Leptin 2. A 6-year-old girl with’ moderate mental retardation with difficulty with her vision. PE shows a fair complexion and malar erythema and bilateral dislocated lenses. She has long, slender hands and fingers. Blood tests reveal her cystathionine concentration is below normal. which amino acid is MOST likely to have an increased urine concentration? A. Methionine c. Alanine B. Leucine D. Arginine : 3. R S-year-old boy bétn of a first-degree relatives was noted to have darkening of the urine when it was left standing. He has a normal sibling, and there are no other medical problems. Growth and development ' to date are normal. Which of the following is MOST likely to be elevated in this patient A. Homocysteine C, Methylmalonate B. Homogentisate D. Phenylpyruvate 4. Which of the following hypothesizes that the physical and functional status of certain region of genomic chromatin is dependent on the patterns of specific histone posttranslational modifications and/or DNA methylation status? ‘A. Genetic code C. Epigenetic code 8. Morse code D. PTH hypothesis §. which of the following is BEST described as being trans-acting? A. Promoter c. CAP site B. Operator D. Repreesor Which of the following statements most closely refletts the actions of the translation factor eIF-4E? Facilitation of the ternary complex binding to #08 ribosomal subunit B. Scaffold for the binding of eif-4A Exchange of GIP for GDP bound to eif-2 Interaction with eif-4¢ in order to bind to the cap structure of the mRNA day-old male newborn develops lethargy, poor feeding,. vomiting, an” , 2 bid Yeopizstions. He was delivered at term after an uncomplicated Disency, His brother died in infancy. He has been-breastfed since birth. serene citrulline concentration. Urine orotic acid concentration mgDs ver 2.1 Continued on Page 2 HYSIGIANS Licensure Examination sundey, March 8, 2020 - 08:00 a.m. - 10:00 a.m. Page 2 SETA STOCHEMISTRY is markedly increased. A deficiency in which of the following enzyme activities is the most likely cause of these findings? A. Ornithine transcarbmoylase . B, Branched-chain -keto acid dehydrogenase . C. Glucose 6-phosphatase D. Medium-chain acyl-CoA dehydrogenase _ Wo 8. Which one of following statements about this stop in glycolysis catalyzed by phosphofructokinase and gluconeogenesis by fructose 1,6-bisphosphatase is correct? A. If phosphofructokinase and fructose 1,6-bisphosphatase are both equally active At the same time, there is a net formation of ADP and phosphate. / : B. Fructose 1,6-bisphosphatase is mainly“active in: the liver in the fed state. . Fructose 1,6-phosphatase is mainly active in the liver in the fed state Which one of the following reactions is unique to gluconeogenesis? A. Phosphoenolpyruvate pyruvate B. Glucose Saphosphate fructose 6-phosphate C. 1,3-Bis-phosphoglycerate 3-phosphoglycerate D. Oxaloacetate phosphoenolpyruvate 10, A patient had recurrent episodes of mild hemolytic anemia. Analysis of these cells shows that they are highly sensitive to oxidative stress likely due to the fact that they contain significantly less glutathione than normal erythroaytes. A deficiency in which of the following enzymes is, LIKELY in these erythrocytes? A. Glutamate dehydrogenase ‘ B. Ornithine decarboxylase C. Cystathionine b-synthase D. G-glutamylcysteine synthetase 11. Although the role of aPC in the termination of coagulation is extremely important, it also serves many additional functions that after the inflammatory processes occurring in the vasculature. The anti-inflammatory functions of aPC are, in part, due to the activation of which of the following? A. Antithrombin III B. Protease activated receptor-1 C. Thrombomodulin D. Tissue plasminogen activator 12. A deficiency in which of the following factors of hemostasis, whose symptoms generally do not appear prior to age 15, wowld manifest’ with recurrent deep vein thrombosis, pulmonary embolism, aiid ¢érebral vein thrombosis? A. Factor VIIT B. Protein Cc A 22-year-old man feels thirsty all the time, urinates frequently, and is most concerned about his weight loss. Glucosa ‘levéls are elevated and has a slight increase in ketones. A diagnosis of Type 1 diabetes was made. Which additional abnormalities is MOST 1ikely to be present .prior to initiation of treatment? A. Increased intestinal gluconeogenesis from muscle-glutamine B. Decreased renal threshold for glucose Decreased gluconeogenesis from alanine cs . D. Decreased concentration of fructose 2,6-bisphosphate in the C. Antithrombin 112 D. Von Willebrand ‘factor 13. liver Continued on Page 3 mops ver 2 PHYSICIANS Licensure examination Sunday, March 8, 2020 - 08:00 a.m. ~ 10:00 aim. Page 3 BIOCHEMISTRY SETA 14. 15. 16. 7. 18. 19. 20. 2° Children with kwashiorkor usually have a fatty live. This ie the result of the . . A. lack of substrates for protein synthesis in the liver ' B. high protein content of their diet C. lack of substrates for gluconeogenesis in the liver D. high carbohydrate content of their diet The base sequence of the strand of DNA used as the template for transcription is GATCTAC. What is the base sequence of the RNA product? (ALL sequences are written 5’ 3" according to standard convention.) A. GUAGAUC cc. cUAGAUG : B. GAUCUAC D. GTAGATC A dS-year-old man was rescued from being trapped under rubble in a building collapse for 4 days. His serum glucose concentration is within the reference range. Which of the following processes is the MOST likely explanation for the maintenance of a normal serum glucose concentration in this patient? 7 A. Protein synthesis ° B. Increased hepatic glycogen storage C. Decreased ketone body formation D. Gluconeogenesis The hormones, epinephrine and glucagon have which ONE of the following : effects on glycogen metabolism in the liver? . A. Both glycogen phosphorylase and glycogen synthase are activated by phosphorylation but at significantly different rates B. Glycogen phosphorylase is inactivated by a rise-in Ca2+, whereas glycogen synthase is activated. €. Glycogen phosphorylase is phosphorylated and active, whereas ‘ glycogen synthase is phosphorylated and inactive. D. cAMP-dependent protein kinase A is activated, wherea: phosphorylase kinase is inactivated. A 7-month-old infant is experiencing feeding difficulty and generalized hypotonia of the limbs. strange eye movements tending to cause the eyeballs to rotate up that occur several times a day were also noted. Lab results indicate significant aminoaciduria and aminoacidemia. Which of the following enzymes is most likely defective in the infant? A. L~amino acid oxidase C. Asparaginase B. Glutamate dehydrogenase D. Glutaminase You are examining a 7-year-old male child who exhibits delayed motor development, over exaggerated reflexes, and spasticity. In addition, the boy has a tendency to chew his lips and fingers often causing tissue damage and bleeding. He also has painful, swollen, and inflamed joints. The most likely diagnosis in this case is Lesch-Nyhan syndrome. Which of the following compounds would be expetted to be elevated in the urine of this patient? A. Xanthine B. Guanine C. Uric ‘acid D. fypoxanthine tips of all eukdéryotic chromosomes? ‘A. Micromere C. Kinetochore B. Telomere D. Chromomere - The disaccharide lactulose is not digested but is fermented by intestinal bacteria to yield 4 moles of lactate plus four protons. \ Ammonium (NH4+) is in equilibrium with ammonia (NH3) in the blood stream. Which of the following best explains how lactose acts to treat hyperammonemia? What is the on of the unusual repeated stretch of DNA localized at the fe ° Continued on Page 4 Tgps Ver 2.1.9 PHYSICIANS L: Examination Sunday, Hexch eee zo20 "06100 a.m. ~ 10:00 a.m. Page 4 , SETA BIOCHEMISTRY a of lactulose results in an eightfold increase inthe ounolality of the gut contents, so that there is more . water for anmonia ammonium to dissolve in, so that Less is : sorbed into the bloodstreal B. Fermentation of lactulose results in acidification of the gut contente so that ammonia diffuses from the blood stream into the gut and is trapped as ammonium that cannot cros back. Fermentation of lactulose results in an eightfold increase in the osmolality of the gut contents, so that there is no more water for the ammonium to dissolve in, so that more will diffuse for the bloodstream into the gut. D. Fermentation of laczulose results in acidification of gut contents so that amnonia produced by intestinal bacteria is trapped as ammonium that cannot diffuse into the bloodstream 22. A S-month-old boy presents vomiting, night sweats, and tremors. History revealed that these symptoms began after fruit juices were introduced to his diet as he was being weaned off breast milk. PE reveals remarkable hepatomegaly. Urine tests were positive for reducing sugar but negative for glucose. what is, the infant MOST likely suffering from? A, Glucose 6-phosphatase deficiency B. Galactokinase deficiency C. Aldolase B deficiency D. Fructokinase deficiency : While studying the structure of a small gene that was recently sequenced during the Human Genome Project, an investigator notices that one strand Of the DNA molecule contains 20'As, 25 Gs, 30 Cs, and 22 Ts. How many of each base is found: in the complete double-stranded molecule? ; A. AR42, Go55, C=55, T=42, C. ARGS, GedS, CH52, TH52~ B. Aw40, G=50, C=60, T=44. -D. Andd, G=60, CH50, TH40. 23. An individual displays an anemic condition and upon molecular analysis is shown to be @ compoumd HbS/HbC. The symptons exhibited by the patient are more severe than those exhibited by patients with sickle cell trait (HbA/HbS) owing primarily to in pai A. increased concentration of HbS B, alteration and increased volume C. precipitation of hbs molecules D. increased concentration of iibC molecules 24 25. The underlying mechanism by which GAGs allow for the formation of gel-like is . A. hydrogen bonding between gag chains B. charge repulsion between gag chains C, covalent cross-linking between gag chains D. charge attraction between gag chains 26. A 43-year-old man is anehic, with a hemoglobin level of 12.2 g/db (normal is 15.5 g/dL). the erythrocytes are microcytic (MCV =°70 £L, with normal MCV = 80-100 £1). Which would MOST likely ba present in’this patient? A, Vitamin B12 deficiency C. Folate deficiency B. Vitamin K deficiency D. Iron deficiency 27. A blood sample is taken from a 40~year-old-man has been fasting completely for a week, drinking only water. Which of the following will be at'a higher concentration than a normal overnight fast? A. Ketone bodies C. Triacylglycerol B. Glucose D. Insulin Continued on Page 5 TQDS Ver 2.1.! Page 5 SETA YHYSICIANS Licensure Braninetion 19.09 an, Sunday, March 8, 2020 - 08: . AEOCHEMISTRY ela 28. A 14-year-old boy with type } > Following insulin IV, glucow: i BEST explains the effect of t in the reduced serum glucose? f A. Activates GLUT by a camp-.«/andent mechanism nfornational. cis we in GLUT €! Promotes GLUT! mobilization © the plasma membrane 5. Increase the Kn'of GLUT¢ for glucose a has a serum glucose 540 mg/dL. ae er eee Ae ee epletion is being studied. Mice are fed lacking * iM the trace metal zinc fbr a poriod of 2 weeks. Analysis of ant GABA neurotranemitter levels and fonction at this tine indicates that GABA, | production is significantly reduced. Which of the following symp! MosT likely to be observed as s result of zine restric Bi Enhancement in the’ ectivity of mxeitatory nerve fibers . Enhancement in reward-driven learning processes D. stimulation of hérmone release from the pituitary d ieoti FAD, which of the 30. As an essential component of the nucleotide analog, FAD, ; following vitamins plays a major role in the transfer of reducing equivalente? ‘A. Vitamin BE ©. Vitamin BL B. Vitamin B2 D. Vitamin B12 31. Which of the following metabolic processes is promoted by insulin? A. Glycogenesis ‘Gluconeogenesis B. Lipolysis tty acid oxidation + 32, Hethotrexate is used as an anti-cancer drug and treatmont for psoriasis and rheumatoid arthritis, This folate analog directly inhibits A. FH2 to FHA C. FH2 to folate B. UMP to dIMP D. Serine to glycine 33. Which one of the following statements about the digestion of lipids is CORRECT? A. Colipase facilitates the binding of bile salts to mixed micelles, maximizing the activity of pancreatic lipase B. Large lipid droplets are emulsified (have their sufface area increased) in the mouth through the act of chewing” (mastication). . Patients with cystic fibrosis have difficulties digestion since their thickened pancreatic secre: jess able to reach the small intestine, the primal lipid digestion ee >. Lipid digestion begins with acid-stable lipases that! primarily use TAG with long to very long-chain fate’ as substrates. : 34. Overproduction of which of the following hormones bya nedBlasn is best diagnosed by an increase in ur. nary excretion: of vanillylniandelic acid? A. Calcitonin c. Epinephrine ‘ae. B. Aldosterone <7 acta M 35. A Aomonth-old infant who suffered bouts of Jalindice andiwee with severe nonhemolytic icterus shortly aftapsmicen ea kernicterus. During the periods of jaundice, be y increased concentration of indirect-reacting bilirubin wi of conjugated bilirubin. The premature death: and early oc in this infant suggest which of the following disorders? A. Acute intermittent porphyria * B. Gilbert syndrome cor dr Bege 6 TQDS Ver 2.1.5 wuerr nase oe, guzy = vorey wr BIOCHEMISTRY C. Crigler-najjar syndrome, type 1 . D. Rotor syndrome 7 ; Which of the following best explains how the brovn adipose tissue newborns generates large amounts of heat Ar Increased availability of fat Mitochondrial Leak of hydrogen tone Increased availability of carn. tine Decreased availability of carnitine 37. A 6-year old boy is suspected with congenital defictency in coagulation. . is first week of life, he bled from the uw circumcision sitet: te had frequent sechysesse ane hamatonas. ie parents are first cousins but denies history of bleeding aésorde Blood workup results showed normal PT, PTT, fibrinogen, platelet” aggregation and von Willebrand. Plasma was incubated w: oer Catt with the clot dissolved in the presence of urea. The deficiency is this patient is . A. prothrombin C. Factor XIIT B. factor vIIr D. factor viz 36. : . baby 38. An infant brought for consult due to the onset of seizures. The exhibited a progressive lack of energy, feeding difficulties, hypotonia, abnormal jerking movements, and difficulty of breathing. These symptom HOS? Likvey tye Sefect in the processes of neurotransmission and are MosT likely due to which of the following disorders? A. Glycine decarboxylase deficiency B. Maple syrup urine disease C. Alkaptonuria D. Hartnup disorder ' 39. Which of the following would most likely be associated with a negative nitrogen balance? A. Consuming a high-protein diet B. Surgical recovery C. Post-starvation feeding + Neonatal development 40. The drug, doxorubicin, is useful in the treatment of lymphomas and breast cancers because of its ability to interfere with which of the following enzyme activities? 2. DNA ‘polymerase C. topoisomerase Ir B. DNA ligase D. uracil N-glycosylase 41. Chromatin remodeling is associated with alterations chromatin remodeling? A. Remodeling is necessary to induce the property: imprinting - Histone acetylation tends to destabilize: chrdina: C. Methylation of guanine residues induces ‘the ‘re >. Methylation of histone hi is sufficient 3 remodeling 42. 2 2 year-old boy, of normal weight and height hae: Blood tests revealed anemia, with microoyts, boy lives in a 100-year old apartment building and hash paint chips. He apparently eats well and takes vitamins is his anemia MOST likely attributable to? A. Heme C. Vitamin B6 B. Vitamin B12 D. Iron Continued on Page 7 TODS Ver 2.1.5 PHYSICIANS Licensure examination 19.99 aim, vie? Sunday, March 8, 2020 - 08:00 a.m om. BIOCHEMISTRY 43. . idosis, i by correction of the ac: lycol poisoning is a Fomovel of any reneining ethylene glycol, and adninistration of an inhibitor of alcohol denyseogentte i oo eet kskaseley Peter! “ Fe Giren ts err e menyiens glycol poisoning by competitively inhibiting conpetitl i Y nol . ADH. As a competitive inhibitor, ethai n: decreases apparent Vmax without affecting Km B. decreases apparent Km without affecting Vmax G! increases cpparent Una without affecting Km D. increases apparent km without affecting Vmax 44. R T-year-old boy with moderate mental retardation is brought to the 45. 46. hetoid hysicdan for a follow-up examination. He has had choreoat! rovements since the age Of 16 months and has been biting his lips and . Eingers for the past 2 months. His older brother has a similar condition. Physical examination shows spasticity of the lower Limbs. laboratory studies show an increased serum uric acid concentration an gn increased urinary excretion of uric acid. itis increased serun acid concentration is MOST likely due to which of the following A. Increased adenosine phosphoribosyltransferase activity B. Increased adenosine deaminase activity C. Deficient hypoxanthine guanine phosphoribosyltransferase activit: D. Deficient xanthine oxidase activity a-l-antitrypsin. (qiAT) deficiency can result in emphysema, a lung pathology, because the action of elastase, a serine protease, is” unopposed. Deficiency of a,AT in the lungs is the consequence of its zetention in the liver, the site of its synthesis. Proteins such as oAT that are designed to be secreted are BEST characterized by which of the following statements? A. They always contain methionine as the N-terminal amino acid. B. They are produced from translation products that have an N- terminal hydrophobic signal sequence. C. They contain no sugars with 0-glycosidic linkages because their synthesis does not involve the Golgi apparatus, D. Their synthesis is initiated on the smooth endoplasmic reticulum. Some types of malignant cells lack expression of fibronectin. The loss of fibronectin by these cells would most likely be associated with which of the following? A. Loss of collagen processing in the extracellular matrix B. Increased adherence of the cells to the basement inembrene C. Reduced integrin-mediated cell adhesion D. Inhibition of matrix metalloproteinase activation 47, Dina, licyear-old, type 1 diabetic patient, forgot to eat after she sel ser cori A B c D f-administered her afternoon insulin, Blood exam showed that her um glucose level is 18mg/dL. Which of the following ‘statements rectly defines the metabolic situation in this patient? + Hepatic glycogen releases glucose in response to insulin-mediated dephosphorylation of glycogen phosphorylase - Phosphorylation of hepatic glycogen synthase proverse glucose incorporation into glycogen — + A reversal of the muscle hexokinase reaction cofverts glucose 6-phosphate to glucose for oxidation in “glycolysis Hepatic glycogen releases glucose through a reverse1 of the glycogen synthase reaction Continued on Page 8 TOUS vereain PHYSICIANS Licensure examination = 10:00 a Sunday, March 8, 2020. — page 8 ‘08100 a.m. SETA BIOCHEMISTRY 48. 49. 50. 51 52. 53: 54. ins 3 amnalian mitochondria contains invert the energy of oxidation- . 1e*e om that. can be Used for the synthesis of ‘This form of energy is most likely an The electron transport chain of mi multienzyme protein comp: reduction reactions into a AIP and for the uptake of solutes. example of which of the following ‘A. Proton gradient only B. Sodium and potassium gradient C. High-energy: phosphorylated ester D. Sodium gradient only vitamin D is deficient, what 2 disorders occur most readily? A. Kidney failure and bent bones B. Rickets and osteomalacia C. Osteomalacia and xerophthalmia D. Osteoporosis and diverticulosis demic, a young In preparation for a trip to Palawan where malaria is en non is given prinaguine es prophylaxis. Soon he develops @ hemolytic condition, The MOST likely cause of the hemolysis is a less-1 level of which of the following? ‘A. Oxidized form of NAD B. Ribose 5-phosphate €. Glucose 6-phosphate D. Reduced form of glutathione : A young boy presents. with a distressful abdominal pain in a hospital. Blood triacylglycerol level was in excess of 2,000 mg/dl (normal = 4-150 mg/dl). The patient was placed on a diet severely limited in fat, but Supplemented with medium-chain fatty acids, Which of the following explains why medium-chain fatty acids are given? A. Enter directly into the portal blood and metabolized by the liver B. Stimulates VLDL production by the liver C. Can be converted into a variety of gluconeogenic precursors D. Are more efficiently packed into serum lipoproteins A hypoglycemia-inducing drug is designed to inhibit dipeptidylpeptidase IV (DPP4) activity and has been shown to decrease plasma glucose concentration and pancreatic glucagon secretion. DPP4 hydrolyzes which hormone when inhibited results in the observed effects? As Prohormone convertase-2/3 B. Insulin C. Glucagon-like peptide-1 D. Glucagon 2 28-year-old male presented with hypogonadism and a¥diomyopathy. Confirmatory test showed that the patient has juvenile, ‘hemochromatosis type 2A, @ disorder from a defect in the gene encoditig’the hemojuvelin protein. ‘The expression of which of the following préteins would likely be unregulated in this patient? A. hepcidin C. hephaestin B. ferroportin D. transferrin receptor The catabolism of hemoglobin . A, results in the formation oF protoporphyrinogen _...» B. involves the oxidative cleavage of the porphyrin ating C. results in the liberation of carbon dioxide» D. is the sole source of bilirubin Continued on Page 9 TQDS Ver 2.1 IYSICIANS Li Examination page 9 PHYSICIANS, Licensure. Examines op aan. ~ 10:00 a.m. 3 ~ wal iate donor of the phosphoryl group 55. Which is the immediate doufucose metebolism in erythrocytes? B Ben ees aehyae Sepnoapnece has been an effective chet which of the following rey motherapeutic agent 56. The anticancer drug, Taxol, presents in the fight against ovarian cancer. ec Of action of Taxo: | the Megteracts with topoisomerase ii preventing its role in DNA synthesis, which effectively terminates replication . B. Interferes with steroid hormone xeceptor interaction with | DNA, thus preventing the growth induction by this class o hormone . ea Cc. Binds to microtubules, which stabilizes them preventing their shortening and interfering with cell division D. Binds to the activated form of the RAS protein which in turn interferes with the signaling cascades involving this protein 57. Heparin is a rapidly acting, potent anticoagulant that has many | important clinical uses. Wnich of the following is an action of heparin? A. Activates prothrombin B. Activates antithrombin ITI . C. Inhibits calcium action D. Decreases prothrombin time 58. Which of the following is CORRECT about the peptide bond? A. Is ionized at physiologic pi B. Is stable to heating in strong acids €. Bas a partial double-bond character D. Occurs most commonly in the cis configuration A 38-year-old man complains of occasional right upper quadrant pain. He had elevated aspartate transaminase (AST) and alanine transaminase (ALT) for the past 3 years. Past medical history includes reflux esophagitis, hypertension, and hyperlipidemia. The patient gained 50 1b since age 18 and consumes less than 5 alcoholic beverages per week. Which is the MOST likely diagnosis in this case? A. Thyrotoxicosis B. Type 1 diabetes ©. Nonalcoholic fatty liver disease D. Type 2 diabetes 59. 60. A 52-year-old woman was diagnosed 2 years ago with primary biliary cirrhosis. She is at greatest risk for becoming deficient in which of the following? A. Bl (thiamin) CE B. B12 (cobalamin) Bec When the malarial parasite invades a red blood cell, its metabolic waste products result in the acidification of the cytoplasm, Which BEST describes the consequences of this acidification on the activity of hemoglobin? ‘A. There is a shift to a more T state conformation B. Formation of carbaminohemoglobin is enhanced C. There is a shift to a more R state conformation . D. Hemoglobin tetramers become less stable and the complex dissociates 61. Continued on Page 10 gaps Ver 2.1 PHYSICIANS Licensure examination = 10106-wsins Page 10 Sunday, March 8, 2020 - 08:00 a.m. BIOCHEMISTRY SETA renal failure from hyperuricemia on the second de therapy for acute myelogenous leukemia. most likely cause of his hyperuricemia? duction via the urea cycle from lysed 62. A 3-year-old boy develops day of cytosine arabinosi * Which of the following is the ‘A. Inhibition of orotic acid pro B. Increased catabolism of purines that are released 1 leukemic cells C. Inhibition of renal tubular transport by cytosine arabinoside Inhibition of uric acid for xenal excretion by ketone bodi jan and as such is concerned about getting 63. A patient is a strict vegetar: sufficient iron in his diet. which of the following suggestion could increase his dietary iron absorption? ‘A. Reassure him that iron in plants is readily absorbed B. Vitamin B12 would help in iron absorption if taken ‘ C. Squeeze fresh lemon juice on spinach salad _ D. Never peel potatoes when preparing potato dish D. 64. A mutation that results in the loss of the formation of the iron response element (IRE), in the 5‘-UTR of the ferritin mRNA will be expected to have what effect? ‘A. Decreased translation of the mRNA in the presence of high iron : B. Increased translation of the mRNA in the presence of high . iron C. Decreased translation of the mRNA when iron is low D. Increased translation of the mRNA when iron is low 65. Which of the following explains the ability to produce lactose in breastmilk of a female with classic galactosemia due to GALT deficiency? , As The enzyme deficient in galactosemia is activated by a hormone produced in the mammary gland. B. Free galactose is the acceptor of glucose transferred by lactose synthase in the synthesis of lactose. C. Galactose can be produced from a glucose metabolite by ‘ epimerization. D. Galactose can be produced from fructose by isomerization. 66. The DNA segment from which the primary transcript is copied or transcribed is called . A. coding strand C, transcriptome B. translation unit D. initial codon 67. Which one of the following statements concerning vitamin B12 is CORRECT? A. It requires a specific glycoprotein for its absorption. B. The cofactor form is vitamin B12 itself. Ci It is involved in the transfer of amino groups. D. Its deficiency is often caused by a lack of the vitamin in the diet. 68. The initial reaction of the de novo synthesis pathway for pyrimidine nucleotides begins with glutamine and CO2 and is complete with the formation of uridine monophosphate. Which of the following represents the rate-limiting enzyme in this pathway? A. Orotate monophosphate decarboxylase B. Ribonucleotide reductase C. PRPP synthetase D. Aspartate transcarbamoylase Continued on Page 11 qons)Ver 2.1.9 PHYSICIANS Licensure Examination —. ini00' aim Page 11 Sunday, March .8, 2020 - 06!00 @.™~ mi BIOCHEMISTRY SETA of 69. Activated platelets stimulate vasoconstriction through the production 7 hich of the, foltoying? c. Prostagiendin # Bi thromboxane B. Prostacyclin d toes. Blood hi erved to have bluish fingertips an 70 Aue Taveeds mild anomie, and child has an inherited erythrocyte | eid jena bis me mutation lea gyzuvate, Kingse def ictertirocyte which helps ameliorate the effects of Sveis in the er ons whae ie the invrease in 2,3-BPG levels due to? to the hosphoenolpyruvate levels lead to tl er sPpnosphoglycerate, forming 2,3-BPG B. The increase in phoiphoenolpyzuvate levels lead to the phosphorylation of !~phosphoglycerate, forming 2) 3-BF . C. The increase in phosphoenolpyruvate levels lead to an | increase in 3-phospioglycerate, which is phosphoryla y ATP to produce 2,3-3PG A. The increase in phosphorylation human DNA 71. WBURGTTT is a restriction endonuclease commonly used to cut nto pieces before inser-ing it into a plasmid. Which of the following 8 most likely to be the recognition sequence for this enzyme? A. AAGAAG 7 AAGGAA B. AAGTTC 72. Which one of the following statements concerning dietary lipid is CORRECT? , A. Fatty acids containing double bonds in the trans configuration, unlice the naturally occurring cis isomers, reise plasma Cholesterol levels. B. Corn oil and soybeai oil are examples of fats rich in saturated fatty aciis. . Triacylglycerols ob:ained from plants generally contain less unsaturated fatty aids than those from animals. D. Coconut and palm oi’s are rich in polyunsaturated fats. 73: A 10-month-old child is jresented at the ER for vomiting and severe diarrhea which began wher the child was fed with cow’s milk. The infant showed signs of failure to thrive, weight loss, hepatomegaly, and Jaundice. Lab tests show elevated blood galactose, hypergalactosuria, and metabolic acidosis w:th coagulation deficiency. These clinical and jeboratory findings are vost consistent with a deficiency in which of the following enzymes? A. Fructokinase B. UDP-galactose uridy.transferase C. Aldolase B D. Glucose-6-phosphate dehydrogenase 74, Born at 28 weeks of gestation, an infant was diagnosed with tespiratory distress syndrome (RDS). Which of the following statements: abour thic syndrome is TRUE? A A. It is a consequence of too few Type II pneumocytdii.. | B. The lecithin/sphincomyelin ratio in the amilotié fwid 4g likely to be greattr than two. C. RDS is an easily treated disorder with low mortality. D. The concentration uf dipalmitoylphosphatidyl-choline in the amniotic fluid is ‘ower than that of a full-term baby. Continued on Page 12 mops ver 2.1. i ation Fusing of cardiac tubes —> Cardia septation -> Development of aortic arch B: Fusing of the paired cardiac tubes -> Heart tubes begin to loop ventrally -> Cardiac septation -> Development of aortic arch i C. Looping of cardiac tubes -> Fusing of cardiac tubes -> Development of aortic arch +> Cardiac septation Fusing of the paired cardiac tubes -> Heart tubes begin to loop dorsally -> Development of pulmonary arteries -> Cardiac septation 4. During auscultation of an asymptomatic child,’ you note that’ his 2nd heart sound is widely split and this split is fixed during all phases of respiration. What congenital heart disease will you MOST likely consider? A. Patent ductus arteriosus B. Ventricular septal defect c. Tetralogy of Fallot D. Atrial septal defect 5, Which drug may be used in the management of acut Ai Gabapentin 900-1800mg in divided bid B, Cycloheptadine 0.2-0.4 mg/kg in divided bid Cc. Ibuprofen 7.5mg - 10mg /kg/dose D. Propranolol 10-20 ml tid ver 2.1-! Continued on Page 2 gps Ve! D: e migraine in children? PHYSICIAN Lice: Mond: ngure Examination lay, September 21, 2020 - 11:00 a-m. - 01:00 p.m. Page 2 seTB PEDIATRICS AND NUTRITION 6. that could a bulging fontanel indicate? Au Bypothyroidiom B. Infant i= crying C) Increase! intracranial pressure D: wormal 7. How is ‘satiety ' y ‘ettmulated? By__+ n Enesetscebec the inueattne. 11 bowels. B. distention 0: stomach or upper sma. disturbance of intestinal disease, C. pain or emoti te of getting hungry. D. feeling and + fe Post Streptococcal : {in years) aftor the initial ‘ hematuria of Acute 8. Persintent microsc. rireist for how long Glomerulonephritis cv presentation? ‘A. Gmonths c. 3-4 BL 1-2 D. 5 9. In ehildren with Conger al Hypopituitarism, which ie normal? A. Skeletal age €. Sexual maturation B. Weight for heiyht D. Linear growth sexual characteristics 10: Precocious puberty is the onset of secondary before what age? As Age 10 in both girls and boys girls and -boys B. Before 10 years in a'yeer in girls and before 9 year in boys C. Before D. Age 12 in girls and age 14 in boys SITUATIONAL situation 1 - Physical Sxamination of the skin of the neonate reveals a lot of Examit these changes may bo needing further transient changes, history taking and physical examination. ments may NOT be TRUE of “Mongolian spots” » \d to disappear within the 11. The following state! re benign and ten ‘A. These patches 4! first year+ B. They ave caused by delayed migration of pi gments from the neural crests. cc. These anthropologic patches mostly appear on Mongolian babies - well demarcated pigmentations on D. They are slate blue, buttocks and back. 12, which one of the following features is NoT a sign of a postmature baby? ‘A. Meconium stained amniotic fluid B. Gelatinous and transluscent skin C. Peeling and parchment-like skin D. Protruding fingernails and toenails stations may disrupt 13, which ones of the following neqnatal manif extremities, face or trunk causing disruption / defect on the skin? "a: Mongolian spots ¢C. Pustular melanosis B. Amniotic bands D. Erythema toxicum i i - i ially if engaged in th ation 2 - The skull of an infant may be, molded especially situation, 7 anal for a long time but those born breech or by cesarean Poction may be well round. mops ver 2-1-9 Continued on Page 3 BEYSICIAN Licensure Examination Page 3 Monday, September 21, 2020. - PEDIATRICS AND nuZRrTTON 14. How does one determine the anterior and posterior fontanels? By__ 11:00 asm. = 01:00 p.m. SET B . A. MRI. c. x’ray. B. Palpation. D. Observation. : 15. The anterior fontanel large if it is more than which normal size? . A. 18 mm C. 32 +/- 5 mm B. 20 +/- 10 mm D. 15 4/- 3 mm 16. Which one of the follow q disorders is NOT associated with a large anter“or fontanel? A. liydrocephalus Cc. Promaturity D! Achondroplasia B. Cranial synostosis Situation 3 - Differing feedim schedules of infants can produce different parental reactions. 17. What will happen if-infant cries because he is hungry and parent offers her breast or bottle? ¢. infant gets full milk A. infant stops crying B: infant is satisfied D. tension dissipates 18. Feeding on demand offers link to which experiences? A. feeding as the pleasurable reduction of: tension B. infant's distress, arrival of parent, and relief from hunger C. unwanted feeding, feeling of fullness Di unrelieved hunger and unwanted feedings 19. Infants who are fed according to parents convenience will LEAST likely experience which’ one of the following? A: increased irritability B: physiologic instability like diarrhea, poor weight gain ¢. late behavioral problens DB: feeding as the pleasurable reduction of tension Situation 4 - An 18 year-old year student developed multiple clear vesicles described as like ‘a dew drop on a rose petal’, noted on her forehead, arms and lower abdomen. This was associated with fever, malaise and myalgia for which she took paracetamol. is the student having? A. Disseminated Herpes c.' Impetigo contangiosa B. Drug reactions D. Varicella could be done to document her problem? A. Do a gram stain c. Request for CBC B. Do a skin biopsy D. Do a Tzanck smear 22. How “ong will she be contagious? ‘Until the last crust had fallen off. until her fever disappears Cc. Two to Three weeks Until there are vesicles. coming out . malaise and headache jtuation 5 - A 16-year old student developed fevers § ‘owed by appearance of pruritic papules to vesicular rashes ‘a rose petal on a foll 204 days after. Lesions are described as like a dewdcop’. The primary care physician diagnosed her as having “Chicken pox". 3, How many days is the incubation period of ‘chicken pox’/varicella ? Be 14-16 c. 30 B. 10-21 D. 7-8 mops ver 2-1-5 continued on Page 4 PAYSICIAN Licensure Examination nday, September 21, 2020 - 11:00 a.m. ~ 01:00 p.m. Page 4 PEDIATRICS AND NUTRITION sere Which sites are the initial appearance of varicella? 24. A. "Back and. trunk €. Buttocks and un) sDuttocks and thighs BI Scalp, face, trunk Bi Upper oxtrenstion 25. What is the usual distribution of the lesions? i ‘A: Caudad c. Genteifugal B. Peripheral B! Gentval/eentripetar high grade fever of around 39.7 Situation 6 - a 10 month-old bab: month-old baby boy develo} C which persisted for 3 days. Once fever started defervescing, a blanching maculopapular rash started to appear on his trunk progressing to face areas. The baby also has rhinorrhea, Congestion, inflamed tympanic membrane. 26. What is the baby having? As Scarlet fever c. Rubella B. Roseola D. Robeola 27s What is the primary organism causing this disease? Ay Human Herpes virus 5 C. Human Herpes virus 7 B: Cytomegalo virus D. Human Herpes virus 6 28. Which is the MOST common complication of this condition? A. Myocarditis C. Otitis media B. Seizures D. Hepatitis rl was brought to the clinic because of multiple Situation 7 - A 6-year old gi umbilicated pearly dome shaped papules and nodules located on her chin, eyebrow and chest. Lesions had been there since start of : summer, and had kept multiplying. A classmate also had similar 29: What is this girl having? A. Multiple milla C. Contact Dermatitis D. Nolluscum Contagiosum B. Impetigo contangiosa 20. What organism in involved in this case? A: Varicella virus C. Papilloma virus B. Pox virus . D, Staphylococcus 31. How is this case managed? A: Electrodessication B. Topical acyclovir C: incision and extraction D. oral antiviral medications Situation 6 - A 10-year old boy and his 8-year old sister developed multiple ‘dtchy papules on their wrist areas, under both armpits, periumbiical, perineal, buttocks, inner thigh areas and between yy especially at night such that their toes. Lesions are very itch: they have a hard time trying to sleep without scratching. are the children having? Cc. Scabies 32. What ‘A. Papular urticaria B. Insect bites D. Contact Dermatitis 33. What medications can the children effectively apply? A. Crotamiton Cc. Permethrin B. sulfur D. Calamine * mops ver 2.1.9 Continued on Page 5 Pays: EEYSICIAW Licensure examination eptenber 21,2020, ~ 11:00 a.m. ~ 01:00 pm page 5 PEDIATRICS ANC NUTRITION 34. Which orcaniam in involved in this case? + Pless c: Sarcoptes scabie B. arthropod mites Di Beabuge Situation uation 9 - A 14 -year old boy consulted the clinic with his mon because of {netially they thought . Gysnosis and defornity of his finger tips. es something to do with his joining the basketball team of nie" school. Bi ty noticed the tips tobe olde: iile"Senogis put hen thoy noticed the tips tobe colder, they Which could be @ MOST useful rapid test for pulmonary functs seful vapid test for pulmonary function? . A. Test for finger clubbing > ~ Bi Arteriel blood gas analysis C. Spizometry Di Venous blood gas studies 35. 36. How does one test for finger clubbing? . .“Place two similar phalanges ‘together , with clubbing , the A ‘window ‘at the nail beds would disappear B. Place anterior aspects of similar phalanges together, endow! would appear if opposing each other, ansoval shaped ‘w: the fingers are clubbed. C: Oppose anterior aspects of terminal phalange: check for disappearance’ and reappearance of an window when repeatedly opposing then tegether Ds Place dorsal surfaces of terminal phalanges of similar Eingers together, with clubbing, the nozmal ‘diamond shaped aperture’ at’base of nail beds disappears. dated with clubbing ? ~ 5 together and oval shaped 37: Which nonpulmonary disease is NOT assoe: A. Smali bowet lymphoma B, Subacute bacterial endocarditis c! Thyroid deficiency D. Toxic Epidermal Necrolysis Situation 10 - For weeks, Ricky was taking in a - while studying. He stays in their bench to ward-off sleep. Garly morning of the day of exams, Ricky developed dyspnea, Siileuity in lying down on his bed, inspiratory wheezing sounds. bn auscultation, he has several wheezes on both lung fields. For years, he has allergic rhinitis. t of food to keep him awake In the 38. what is patient having? Food hypersensitivity rxn As B. Insect bite reaction(rxn) C. Foreign body obstruction D. Bronchial asthma 39. Waat would be the BEST method to come out with the proper diagnosis? A. Spirometry Cc. History and PE D. Chest xray B, Skin tests S$, which may NOT be TRUE of ‘wheezing"? ‘on auscultation of the lung: CHF produces wheezing. 40. ‘A. Peribronchial edema in B. Whaezes suggest airway obstruction. | €. wheezes can be caused by narrowing of small aizways- D. Wheezing is mostly Bronchial Asthma. situation 11 - A 10- year old boy consulted the clinic with his mon because of halitosis, chronic sore throats, foreign bedy sensation and history of expelling foul-tasting and foul-smelling cheesy lumps- mops ver 21+ Continted on Page 6 PEYSICIAN Licensure Examination jonday, September 21, 2020 - 11100 a.m. ~ 01:00 p.m. page 7 PEDIATRICS 2ND NUTRITION ser B ased activity of the adrenergic nervous system C. Reduced contractility of the myocardium D: Diminished peripheral vascular vasoconstriction 49. Besides cac, electrotytes, BUN, Croatininey hepatic enzymes, what other aboratory workup should be requested for the pattons? ‘ ‘A. Glucose tolerance test | C. Fasting lipid pane) . Bi Urinalysis D! Fasting serum glucose ituation 11 - patty, 8/F, came into your clinic for fever and episodes of she also complains of on jerking of bilateral’ upper extremities. ‘also neck pain. and off headache, photophobia and 50. During your neurological examination, you noted your Kernig sign. What is a positive Kernig sign? Ao BiSxioa of the hip 907 with subsequent pain with external patient to have rotation of the legs Be Feats emo the nip'907 with subsequent pain with extension of the legs - A cs SektSn SF tne hip 907 with subsequent pain with interna? rotation of the legs i D. Flexion of the hip 907 with subsequent pain with flexion of e legs 51: what is No? considered a sign of meningeal irritation? A: Nuchal Rigidity c. Kernig sign B. Brudzinski sign D. Fever 52. which finding is suggestive of increased Intracranial Pressure? ‘A. Hypertension C. Decorticate posturing B. Papilledema D. Nuchal Rigidity 5 absent from school because of frequent severe, with throbbing Situation 15 - 10-year old boy is alway: and vomiting. He headaches, sometimes moderate, sometimes quality. His headache is associated with nausea Siso becomes sensitive to light, and sounds. 53. why is it important ‘to treat patients especially those with headaches gexureing for more then 15 days? Because it is important to —_~ ‘A. prevent restrictions on social activities. B. block transformation to chronic daily headaches. C. prevent further school absences. D. prevent limitation of home activities. 54. Migraine may have associated aura, the typical ones include which symptoms? A. Tinnitus c. Photopsia B. Motion sickness D. Abdominal 55. How long will the aura associated with migraine last? ‘A. Less than 60 minutes but should not be more B: More than 60 minutes c. 15 minutes at least D. Longer than 5 minutes but less than 60 minutes old son swallowed a pointed he doctor told her to situation 16 - Mother reported that his 2" -year popsicle stick while eating his popsicle. 1! assees weekly. 56. which circumstance is NOT an indicati ‘Ac child develops signs/ symptoms o: g abdominal perforation owed no movement within 48 hours on for surgical removal? £ abdominal obstruction B. Pendin c. Foreign body sh mops ver 2.1.9 Continued on Page 8 PHYSICIAN Licensure Examination page 8 Monday, Sey jonday, September 21, 2020 - 11:00 a.m. - 01:00 p.m. PEI PEDIATRICS AND NUTRITION SET B Ds Foreign body fails to progress for several weeks 57. which shou2d be avoided? . Antipyretics c. Analgesics BL Antidierrheals BL Gathareies 58. Surgical removal is indicated for the following ingested forgtan eels which is the ZBAST Likelihood of complications 1f not Temoved surgically A. Water absorbing polymer beads B. Single magnet) G: drug, body packings D. Lithium battery gency room after incidental Situation 17 - Melody, 16/F, was brought to the emer: {ingestion of Dishwashing Detergent after fighting with hex boyfriend. She was rushed to the hospital an hour after odes and epigastric pain. following ‘several vomiting epis Ingestion of caustic substances results to the following EXCEPT for B. Necrosis c. severe gastritis B. Perforation D, Stricture Formation 59. ccidental ingestion of liquid alkali 60. What is the often produced after a substances? 7 A. Stricture Formation B. Coagulation Necrosis C. Protective “Thick Eschar D. Deep Liquefaction Necrosis 61: what is the recommended acute treatment for caustic ingest: A. Neutralization B. Dilution by water or milk C. Gastric Lavage D. Induced emesis ion? jour clinic for fever and yellow Situation 18 - Mario, 17/M, came to ¥ Fever has been on and off for the past discoloration of sclera. Geek with generalized malaise and occasional vomiting. jepatitis A? Ycomatic for younger children. B. Hepatitis A is more often an icteric illness- B: fobatieie a virus is responsible for acute hepatitic onty: D. Hepatitis A is distinguishable from other gastroenteritis. § B Virus is present in high concentrations in the following 62. What is TRUE regarding ‘A. Hepatitis A is symp’ 63. Hepatitii EXCEPT for . c. Saliva ‘A. Blood B. Serous exudates D. Serum 64, What is NOT TRUE regarding HBSAg? at aise of HBSAG closely coincides with the onset, of symptoms. Bs Rise iotence of HBsAg beyond 6 months defines chronic infection state. " Cc. HBSAg is found in almost all persons infected with HBV. 5: aasag is the second serologic marker of {infection to appear i ini ied by his 19 - A 17-year old boy came into your clinse unaccompanied Bitoe art A Mobed the appearance of a vesicle on his penile Chosocial history revealed that lassmates + parents. "ther probing in the psy . Further probli + ee viel, es Sehtton with some of his gps ver 2-1+! fe engaged in sexual experime continued on Page 9 FHYSICIAN Licensure examination Page 9 Monday, Se} ‘Ye September 21, 2020 - 21:00 asm. ~ 01100 p.m PEDIATRICS AND NUTRITION ser B Upon. fur} jpon further evaluation the vesicle was noted to be tender to touch and inguinal lymph nodes were palpable. =~ y Transmitted Infection among 6s. S+ What is the Host common ulcerative Sexial) A. Syphilis é Cc. Genital Herpes ] Agute Genital Ulcers D! Chaneroiad ost common ulcerative Sexually Transmitted 66. What organi ees thil anism cau: 8 MK Infection among adolescents? A. Chlamydia trachomatis C.-Treponema pallidum Herpes Simplex Virus D. Haemophilus ducreyi 67. If the vesicle in the patient turned out to be ainless, what would be . Ee MSs Tikely diagnosis? ® ‘ , ytococcal pharyngitis, Situation 20 — After @ ménth from having 2 strep! ‘inmy developed hematuria, edema, hypertension and oliguria, His primary physician said he has Poststreptococcal . 8 year old Glomerulonephritis. 68. Which manifestations is LEAST likely a sign of having risk of encephalopathy? A. Blurred vision c. oliguria D. Severe headaches Bi New seizures © st Likely a sign of pulmonary edema and heart 69. Which manifestation is LEAS failure? - ‘As Orthopnea c. Respiratory distress B: Fever D. Coughs 40. which age group in years is Poststreptococcal GN most common in childzen? A. 3-5 c. 5-12 B. 12-15 D. less than 3 ted in the outpatient department for sudden ed she had a bout of Situation 21 - Carmen 12/F consul! onset of tea-colored urine. History reveal onset erost 2 weeks prior that resolved without any medical: Soe eee pertinent findings in physical examination include an cyeettea blood pressure at 130/90 with note of facial edema. h is NOT a characteristic of Acute Poststreptococcal 71. whic Glomerulonephritis? A. Edema c. Microscopic Hematuria D. Renal Insufficiency B. Hypertension 72. which laboratory finding is expected to be found in this case? A. C4 is reduced C. C4 is elevated B. C3 is reduced D. C3 is elevated 73. what BEST diagnostic tool confirms prior streptococcal throat infection? ‘A. Positive Urine Culture : B. Positive Throat Culture Rising Anti-DNAse B titer Cc. Rising ASO titers D. consulted the clinic because of delayed onset of 22 - 16-year old Amy, y Y" CGbjective complaints. The, rest of her ubic hair development she had no other su ‘unremarkable, Her breast and p\ v tage 4. The rest of her physical examination finding TODS Ver 2.109 Continued on Page 10 situation menarche+ history was were Tanner § REYSICIAN Licensure Examination ¥, September 21, 2020 - 11:00 a.m. ~ 01:00 p.m. SET B PI EDIATRICS AND NUTRITION were normal. 74. What is TRUE regar i y What te trys regarding the definition of Primary Ar By age of TT years. c. For more than 3 months. 1 Per "2oce than'a month. Ds By age of 16 years. 25. What laboratory finding would suggest primary gonadal faiture? Ay Decreased level of Luteinizing Wormone (Li) B. Decreased level of Prolactin 8: Biecatea evel of thyroid Stimulating Hormone (Ten) §; ELSVaESS iSveie or Follicie stimulating Hormone(FSH) 76. In the approach to clinical evaluation of amenorrhea, what KEY nthe approach £0 cla'be performed first regardless of history OF : sexual activity? ‘A. FSH Measurement c. Pregnancy test B. LH measurement D. Pelvic ultrasound or breast enlargement. Situation 23 - Max, 14/M, came to the olinic £ ‘the changes about 2'months ago. Upon further examination you toe sence hic right breast is larger than his lef: ove, He noted that the breast is tender on: occasions: ‘His Body Mass Index per age is within the normal range- 77. What is TRUE regarding True Gynecomastia? SES eee eee ce adipose tissue in the area of the breast In pre-pubertal boys- B, Presence of palpable fibroglandular mass concentrically below the, nipple and areolar region c. Presence of palpable fibroglandular mast surrounding the nipple and areolar region p, Rives ction of adipose tissue in the area of the breast in overweight boys Amenorrhea? No He noted pubertal Gynecomastia? tata hedications that decrease levels mulation by maternal n = androgen action 78. What is TRUE regarding ‘A. It is caused by exposur of androgen. . tt is the resuit of the normal sti estrogen. c. Te is causéd by imbalance between estroge! in the breast. pe an ene oeete exposure to estrogen during the course of pregnancy: what is TRUE regarding the course of Pubertal Gynecomastia? 79. Te rarely persists longer than _ + A. 12 months. €. 24 months. B. 18 months. D. 6 months. situation 24 - Annie was brought to her doctor's clinic for Penicillin TASections due to her Rheumatic Heart Disease” ‘That afternoon, injert eg started to become tender and chopped Her eyelids became painful and eroded. . 80. what else in her body areas should Fe examined ? ‘A. Soles c. armpits B, Beneath breasts D. Perineal areas gi. what is Annie developing? ‘a. systemic Viral Infections ermal Necrolysis B, Toxic Bpid Cc, Drug Reactions “gqps ver 2-1+ Continued on Page 11 PR SICIAN Licensure Examination page 11 Monday, Se} nday, September 21, 2020, ~ 11:00 a.m, ~ 01:00 pom pep se IATRICS AND NUTRITION = D. Stevens Johnson Syndrome 82. What can 5 hat can pe given st the early Senge, 45,508 development of this problem? A. Inmunooduiacors ic. Antihistamines ] Systemic corticosteroids D. IV Fluide Situetion 5 — A 15-year-old high school student, reviewing, £05 208 exaney tion 755 tha clinic because of presence of scaly exytnenscovs fe noticed this to be a back of his ear Je under stress with an upcoming @xans- plagues cn his scalp, an‘ Sceursing every time he 83. What is this patient “having? ‘A: Atopic Dermatitis ¢. Psoriasis B. Seborrheic Dermatitis D. Contact dermatitis 84. what organiem is commonly involved in this condsetent A. Candida albicans @. pityrosporon ovale B. Staphylococcus aureus Di Demodex mites 85. which one ie NOT a precipitating factor of this condition 7 A. Colder months, ic. Food D. Illness B. Stress mosed with Hansen's disease Mtches noted on his fbacillary leprosy y and Rifampicin- bluish Situation 26 - A 16-year old boy was diag ;tuberculoid type because of hyposthetic P back; He was prescribed medications for pave consisting of Diaininodiphenyl Sulfone(depsore, Week after intake, the patient manifestes wich disccloration of his lips and finger tips a \d to the patient? 86% What happene ‘A. He has fixed drug erupticn. 3B. He has G6PD deficiency. C. He has pyruvate kinase deficieacy- D: He has iron deficiency anemia. 87: What is the triggering factor in the patient's manifestations? A. Dapsone and ‘Rifampicin. Cc. Hansen's disease itself B. Dapsone D. Rifampicin 88. Besides avoidance of the trigger factors, how could this condition be treated ? ‘A. Splenectomy c. Treatment of infection B. Blood transfusion D. Folic acid supplements situation 27 - A month prior to congultation: 19 year-old Nina, obese, develope t Stiple erythematous petechise over her thigh areas. Her lesions progressed downward to her legs, bilaterally, with increased size preging into hemorrhagic nodules. The initial impression was cutaneous vasculitis. 39, which of the following factors is NUM ¢ cause of vasculitis? A. Iafections Cc. Medications B. Smoking D. Chemicals 90. If the lesions start to progrsn® to the abdominal areas “and Upper Zferenities, which organ should be examined? A. Liver c. Kidney B. spleen D. Ovary ops Ver 2-1- Continued on Page 12 PHYSICIAN Licensure Examination Monday, September 21, 2020. ~ PEDIATRICS AND NUTRITION 91. Patient ie havi A 6 having multiple joint padi urine. Of the following laboratory results, which one wou: to be elevated ? Platelet counts Erythrocyte sedimentation rate cr D. Situation mid thigh area. No history of trauma 92. An Xray of the ri ght femur showed primary lytis periosteal reaction , an "Onion peel appearanc what is the MOST plaus: C. Osteosarcoma swelling ‘A. Reticulum cell sarcoma White blood cell count ANA quantitative determination gia -old boy consulted the clinic wi er, weight loss, pain, swelling, and tendern but patient had’ limitation of motion. ¢ bone lesion with je" and soft tissue 28 - A929 y BL Ewing Sarcoma A B 94. what, A. BL ce D. Situation 2 noted a white pupillary reflex: instea examination her left eye. is the current preferred treatmen! Radiation therapy 11100 asm. ~ 01:00 p.m. Page 12 7B SE" ible diagnosis? D. Osteomyelitis abdominal pain ant 4 blood in the ld be expected ‘th his parents because of over the right 93. On Immunohistochemical staining ; which of the following will show ifferentiated small round celi of neural origins? Neuroblastoma ‘c. Lymphoma Rhabdomyosarcoma D, Ewing Sarcoma t of this case? PY Surgical reseétion and Radiation Surgical resection and Chemotherapy Multiagent chemotherapy you then referre for probable Retinoblastoma. 95. As Be 96: Hyphema Strabismus involvement? orbital cr Skull Radiograph A. Be 97. what A. situation of hepat. are symptoms of 4! 98. what A. Bi 99, What Ae Be Cc = TRUE regarding treatmen charger tumors often respon al includes preserving vision and eye itself. 1 of treatment is always cure. still be required for responsive tumors. ulted the clinic with his parents beca Associated with this abdominal pain. agent. ‘The primary go ‘The primary goa ‘nucleation may 30 - An 18 year-old boy consi Ze mass with abdominal distention. norexia, is the patient having? which imaging modality will allow for ¢. orbital ULtrasonography Hepatoblastoma viral Hepatitis condition is MOST commonly associated with t Galactosemia storage disease Glycogen Biliary cirrhosis nitation of Retinoblai What is the COMMON initial presel C. Leukocoria 9 — Jamie, 4/F, was brought to your clinic for well child visit. d of a red reflex during d her to a specialist stoma? D. Orbital Inflammation D. Orbital MRI tof Retinoblastoma? weight loss and Continued on Page 13 d to single chemotherapeutic Cc. Biliary cirrhosis D. Hepatocellular carcinoma umor ? You better evaluation of optic nerve use mops ver 2-1-9 PHYSICIAN Licensure Examination Monday, September 21, 2020 - 11:00 a.m. - 01:00 p.m. Page 13 PEDIATRICS AND NUTRITION ser 8 Ds Hepatitis A or c infection 100. Liver enzymes may be normal but there is an elevation of the __-in approximately 60% of children. A. ScoT/scpr c. Unconjugated Bilirubin B. Transaminases D! Alpha Feeo Protein (AFP) WARNING: seen D Oe Failure to submit your Test Questions (Complete) set will cause the cancellation of your Test-Results for the subject, Republic of the Phi1i Seat Mors __ PROFESSIONAL REGULATION County OM: qauuay: IISSION 7 Puysicz BOARD OF MEDICINE Sunday? *8_icensure Exam: lay, Sep Exgmination : ==~2_September 20, 2020 : 11:00 a.m. - 01:00 p.m. SURGE: oP: SORSERY §_ OPHTH, ITHALMOLOGY,, OTOLARYNGOLOGY & RHINOLOG ser B Instr Surrecones MacfTO0Gs She Setes® eves f7E 00 shat tthiolgs STR ICTLY NO ERASURES ALLOWED. MULTIPLE csorce 1s Bleedin ng after massive blood transfusion followltg* conditions Extese 9 fusion can be a result of the + dilutional coagulopathy Be i pathy C. fibrinogenesis hypofibrinogenemia D. platelet dysfunction 2. a 60- postasperacte, male patient underwent exploratory laparotomy. 24—hours Peeccoperatively, he complained of right, chest pain. ECG is normal, “gponds negative. Which is the most likely problem? eumonia C. Volume depletion B: Myocardial infarction D. Atelectasis 3: "Never events” in surgery includes __- A. surgical site infection Cc. septic shock D. retained surgical item B. post-operative bleeding 4: Prior to surgery, the resident on duty ordered for activated partial Identify the coagulation factor being thromboplastin time (aPTT). tested. A.V c. x B. VII D. VIII The surgeon’s knot tie __. verhand throw ‘A. begins with an overhand throw then an o followed by a standard square knot throw. B. is stronger that a standard square knot. C. begins with a double overhand throw followed by a standard square knot throws. D. begins with an overhand throw followed by underhand throw ery, the wound is preferably Select the condition wherein after the surg Closed by secondary intention. A. Breast biopsy B. Myoma uteri c. Cholecystitis D. Perforated diverticulitis advantage of monofilament sutures over multifilament su I. It encounters less resistance when passing through tissues. I sms causing infections. rbors less organi : II, Tt ha €88 fensile strength even when crushed or crimped. tures [IIs It has grea A, II, III Va B. I, 1, TL D. I, IT tops ver 2-1. Continued on Page 2 Paysz SengzCTAN Ls SURGERY SURGERY op HTHALMOLOGY, OTOLARYNGOLOGY & RHINQLOG 8. 1o. als 12: 13. 14s 15. cen, P Sopecensure Examination ember 20, 2020 - 11:00 asm. ~ 01:00 p.m. Page 2 ser B Which of THEY, OF the sondowing activities yil1 NO contribute significantly in s tmprovenent, of puiponary function post-oporatively? Ry Deep breatn exercises G: Incentive spirometer 'y mobilization D. Early feeding nitude of bacterial load at the time of Sith a clean wound. Based cn ti he presumed ma Pai procedure eesociated Surgery, select the surg ©. ferniorraph: De ‘raphy Debridement of nectotizing fasciitis Prolonges tl ged course of antibiotic is recommended for the following conditions _. I, osteomyelitis IT, post-operative Ur + prosthetic infection A. I, Ir B. I, II, IIx c. 1%, 2Ir D. Ty IT sfusion complications. Select the INCORRECT n Gomplications enumerated fe with anti-histamines ‘transfusion, increase The followin i wing are blood trant response to the blood transfusior S: poneeeia reaction - pretreatmen' : e lung injury - eateeion jury continue blood c. Hemolytic reaction - stop transfusion immediately + Hemolytic resctsovtic reaction ~ pretreatment with acetaminophen which of the following suture characte: i, pliability to allow ease of handling and kno’ TI. uniforn tensile strength Tyr. suture material should be resistant to infection A. I, ID c. II, IIT De I, I, TT . Be, HE a of respiratory failure in a istics are ideal ? ¢ security The following are manifestatio post-operative patient EXCEPT + st-opebiratory rate bbove 30, breath/minute B. elevated tidal volume. Cc. paco2 > 45 mm Hg D: Pao2 < 60 mn Ag oT indicated ncillary tests is MN ‘imonary gj; laboratory and ai Sa eetion of a patient with compromised Pu c. Arterial Blood Gas D. Pulmonary Function Test In a trauma patient's initial survey and evaluation, the following conditions should be identified. These life-threatening conditions warrant immediate attention to decrease mortality __- which of the followin in tae pre-operative function? ‘A, Chest Radiograph B, Complete Blood Count r. cardiac tamponade . tr. Massive hemoperitoneum III: unstable pelvic fracture As I, 12 c. I, IL p. II, It D. I, I, II ops ver ? continued on Page 3 PRYsy Sunday CIAN 1, P SopeSeneure Examination er 20, 2020, = 11100 a.m. ~ 01:00 p.m. Page 3 SET_B SURGERY & SURGERY ¢ 01 PRTHALMOLOGY, OTOLARYNGOLOGY & RHINOLOG 1s. 17. 18. ig. 20. 21. 22: 23 + Which ° 2£ the following is NOT a goal of damage control surgery (DCS) in abbrev rreviating trauma laparotomy? + To limit BL enteric content spillage. To make sure that all trauma injuries are surgically og, Sottected. §; 20 preveni the bloody victous cyte. Eontrol surgicel bleeding while preventing ischemia. To Lim * deerme morbidity during the operation, assessment of bleeding risk is termined prevoperatively in the following patients EXCEPT nti-coagulant therapy . B. Hemoglobin of 10g/di + Liver and kidney dysfunction D. Pre-existing anemia In the criticall; ically i11, post-operative patients, stress-related hyperglycemia is die to the increase of the following hormones EX CEPT Glucagon Thyroid Stimulating Hormone Epinephrine Glucccorticoia e the general level of The Glasgow Coma Scale is used to describ: consciousness in patients with traumatic brain injury and to define ing are components broad categories of head ii Which of the follow: broad categor njury. Which of the foll I. motor response II. pupillary reaction to light IIE. eye opening A. II, III c. I, IZ, Ir B. £, It D. I, IIT In general surgery, which of the following is NOT necessarily monitored in the recovery room? A; vital signs c. central venous pressure B: fluid balance D. distal pulses. s of a mass on her anterior neck area. Tess, a 15 year old girl, complain Biopsy revealed thyroid’ cancer. Which of the following is the MOST probable histologic type? ‘A: medullary thyroid carcinoma B. follicular thyroid carcinoma ¢. papillary thyroid carcinoma D. anaplastic thyroid carcinoma Radical mastoidectomy is performed for __. A, acute otitis externa bhronie otitis media with central tympanic membrane Bs perforation. c. acute otitis media C: Afgdle ear and mastoid disease with cholesteatoma. ror cancer within the oral cavity, a composite resection means excision of the following structures EXCEPT .__- «neck dissection j 2. Nermillion surface of the lip jgnant mass C. mattdon of the mandible mops ver 2-1-9 D, portion ©: continued on Page 4 Paysrcray 5ui NW Lice: nday, Squ{Sensure Examination er 20, 2020 ~ 11100 a.m. ~ 01300 p.m. Page 4 SET B SURGERY & OF: ‘HTHALMOLOGY, OTOLARYNGOLOGY, & RHINOLOG 24. Which with mente, following dis the better predictor of survival in patients : ‘ary thyroid carcinoma? a Serun calclun Seve . oid stimulating hormone Level €: caleftonin doubling, time D. care: as Careinoembryonic antigen + Post-o; Post-operative radiation therapy for malignant neck mess ds indicated x the following BXCEPT : Service lymph nodes Positive for metastasis st tH OF more pb, Limph node levels. | extra caprular spread of cervical lymph node metastasis €: 12 NO MO cancer : {more than two cervical lymph nodes contain metastasis 26: Which of the following is NOT an indication for the surgical removed of @ nodular goiter? Ae Substernal extension + Presence of symptoms of pressure . C. Hashimoto's thyroiditis P + Anterior neck mass suspicious for malignancy 27: Which of the following contributes to the development of astroesophageal Reflux Disease (GERD)? Is Defective Lower ‘ 2: Bs I, Defective Lover ‘Esophageal sphincter (1 S) III. Dysphagia AL IE, 13t c. 2, 101 B. I, Ir D. iy Et, XIE 28: In-early staged tracheal neoplasm, the treatment of choice is A: radiotherapy C. hormonal therapy B. surgery D. chemotherapy 29. which of the following findings indicates incurability of lung cancer? A: Recurrent pneumonia B. Tuberculosis . C. Clubbing of the digit D. Left vocal cord paralysis stage lung 30. what is the primary management of a patient with an early cancer? ‘A. Hormone Therapy c. Radiotherapy D. Chemotherapy B. surgery ing the pulmonary apex. In A Pancoast tumor is a malignancy affect: omplains of pain and weakness of the progressive cases when the patient c Pegies of the arms and hand, there is compression of __ A, phrenic nerve C. superior vena cava B, brachial nerve D. subclavian artery 32, Kyle is a 25 year old patient who arrived in the emergency room with Ryl¢ steral chest pain and shortness of breath. Physical examination and Sine presence of pnewmothorax. According to the patient, ff the following is the most ~ x-ray verifie Y first episode. Which o this is the opie riate treatmen' approproracotomy and pleural abrasion A. Tigertion of chest tube drainage of the pleural space 2; preurodesis with tetracycline injected in the pleural space ees and daily monitoring with chest radiograph Dp. observat Continued 9n Page 5 31. mops ver 2-1-9 Paysy, Cra Sun iN ey, “Sepecensure Examination 20, 2020 - 11200 a.m. ~ 01:00 p.m. page 5 sere SURGE: SERGERY & 0} PRTHALMOLOGY, OTOLARYNGOLOGY & RHTNOLOG 33. In the , to fhe surgical closure of Patent Ductus Arteriosus, care m avoid vhich of the following nerves? terionue, care must be token B. Phrenic nerve §: Slossopharyngeal norve jecurrent Laryngeal nerve od with pneumothorax, loft. The Gated by the medical intern EXCEPT 34. Year old male patient is .diagnos owing situations should be apprec. Shortness of breath nese on poreussion of the left chest Chest pain Decreased breath sounds on the left chest oh of the following differentiates a simple pneumothorax from a tension pneumothorax? Bs Widening of the mediastinum ing of trachea towards the opposite side of the c, Pheumothorax. + Presence of shortness of breat BI Rypoxis and chock OF Prenth 36. Which of the | ch following h: 0 0 ni Which of the following histologic types of lung cancer is least A. Squanoug cell carcinoma C- small-cell carcinoma + Clear cell carcinoma D. Adenocarcinoma 37. A 60 year old, chroni¢ alcoholic developed cirrhosis. Physica? 1g Race in the right lobe of his liver. Blevated agnosis? exanination reveale alpha fetoprotein level. A. Hepatic adenoma B. Hepatocellular carcinoma C. Metastatic carcinoma of the colon D. Focal nodular hyperplasia Which is the most likely d nerable to perforation? Which explains why the esophagus is vul tes retention of gastric 38: ‘A. Positioning of ‘the esophagus promo acid. Bi The esophagus is' easily eroded by gastric acid. C. Esophagus lacks a serosal layer. D. Esophagus is resistant to acid. 39, Juanita, a 60 year old woman consulted for second opinion regarding her upteernal chest pain and vague upper abdominal discomfort for the last supst ths. The pain was not relieved completely by proton pump inhibitor therapy. Endoscopic study was unremarkable, How will ‘you proceed in the management of T.J.? A, Psychiatric evaluation c. Cardiac work-up B, Barium esophahogram D. Manometry 40. Jun is in the ER diagnosed with acute abdomen, in the pre-operative Management of Jun, select the CORRECT statement. IR, Jspecinen of blood for cross matching should be sént whenever urgent surgery is antic ‘pated. Phite blood count and differentiated formation. hematocrit, ‘@ highly informative in: B, Hemoglobin, i dmission ar he abdomen is done in all cases. count done on a c, Plain radiologic test of t c Bp, Angiography is indicated if there is suspected on-going bleeding: 6 mops ver 2-1- Continued on Page ERYSICTAN Licensure examin: a.m. m : re Examination ® Sunday, September 20, 2020, ~ 11:00 a.m. ~ 01:00 p ° ser B SURGERY oP: RTHALMOLOGY, OTOLARYNGOLOGY & RHINOLOG 41. Select ct the cause o: ACE the cause of henatomesis that will require emergency management. C. Esophageal varices BL Mallory weiss tear $: Geckritts on, severe abdominal 42. Acute abdom Deubegetomen te 2 condition, pregensing with sudde Foghize urgent sargery eee tnan 24 house the folowing diseases bleeding : B. ischemic bowel S: perforated viscus 43. Which of th . 1 follow! tpstotony for lowing conditions wilt wor benefit from exploratory » Bowel os =e A: Bowe? perforation c. Mesenteric adenitis, ruction D. Appendicitis 44: cullen’ fatty eects is the superficial edema and bruising ‘dn the subcutaneous tty, tissue around the umbilicus which can predict + As Route nephrorithiasia C. Acute pancreatitis e@ appendicitis D. Acute cholecystitis ed in patients with 45: Which of the followi ease jfellpeing should aleo be monitor . Arterial oxygenation c. Renal function test D, Blood ph B. Electrocardiogram © cancer Mi ose ang Jose is a 65 year old male diagnosed with pancrsats: Which of the following 46. involving the head of the surgical incurability? aids ‘A Sudden onset of diabetes C. Palpable mass B. Couvoisier’s sign D: Jaundice a7: which of the following anatomic structures is NOT inctaced in the n of a gastric carcinoma? tive for resectio! th adjacent uninvolved stomach surgical obje! A, Tumor wi B. Duodenum c. Pancreas D. Regional lymph node was brought to th eevee abgominal pain. While in the Fi the {'_ not to move nis abdomen. whic! ‘a diagnosis of a lays completely stil signs and symptoms does NOT correlate with C. Involuntary guarding ‘Ai Washboard abdomen B. Absent bowel sounds D. Bradycardia sociated with carcinoma of following symptoms is as} eas? e emergency room complaining of patient looks unw 48: Paul; 34 year old, 49, Which of the and tail of the pancr! A, Jaundice B. Migratory thrombophlebitis a C. relief from pain when sitting position with the spine flexed s sign p. Courvoisier’ 50. the definitive managenent for cholangitis is ni broad spectrum TV antibiotic @ resuscitation Bp, flui , * cholecystectony ; c cnsoecupic biliary decompression continued on Page 7 ops ver 2-1 will indicate jell and ih of the following cute abdomen? the body PHYSICIAN Sunday, Sopeoereuts, Examination ial 7 2020 = 11:00 a -M. ~ 01:00 p.m. Page 7 SURGERY & cpuny Senet § OPHTHALMOLOGY, OTOLARYNGOLOGY & RUTNOLOG ser B stile goal of fo, Pancreatic cancer is tr és si SEcpesering pueecacia atgtutten gppusesse sever 8S) 8 52. The ai gic type of ch 5 Paguandu SeS2°Bse tteeoF cpotenetocertaane 2 53. Consexvats Perfosned on a patient wich “atsents with Powel obstruction can be « strangulated hernia B. paralytic ileus 51 Bowel perforation 54. Which of Brsch Of the following is wor given to a patient during bowel, A Mati roertor t© Surgical resection of bowel? : Bhema 7 . Laxatives D. Solid food mea) ZSUISEE SceeAgbes withthe cavetopmant of colon cancer azo the A. ulcerative colitis 4 B: hyperplastic polyp 5 Crohn‘s colitis + inflammatory bowel disease Which ‘ ch of the following are the diagnostic modalities used fo! 56: diagnosis of lymphedema? : Plain radiograph II. Lymphoscintigraphy TII. Dupplex ultrasound ~ ca c. I, 1%, 1It 22, Ir D. II, Iz 57. ‘The most common cause of mechaiiical bowel obstruction in @ 45 year-old post-hysterectomy patient is __- A. colon cancer . €. diverticulitis B: volvulus D. adhesion f the anal canal is often associated 58: Which of the following conditions 0: with a synchronous GIT adenocarcinoma? A. Bowen‘s disease B. Buschke-Lowenstein Tumor Epidermoid cancer of the anus D: Perianal Paget’s disease, hich of the following laxatives, when used in bowel preparation for surgical resection of bowel, produces hydrogen and methane gases that coe aegplode when electrocautery is used? A. Magnesium citrate Cc. Mannitol D. Bisacodyl B. Castor oil . Which of the following statements is FALSE regar = he fonal pain that was originally colicky 1 obstruction that became const 59. jarding bowel obstruction? in a patient tant or worst a may be Bs Any 2! Stapartod of beh SHE Ggylcian the tebe B Srctosed {0p powel obstruction may be manage conservatively | : o. Sanicesetog ok RuiMS Suttle Sc ose cm prevent bowel resection Continued on Page 8 gps Ver 2+1+ PRYSICIAN yj Lic Sunday, SepeceP8tre Examination mb Page 9 SURGERY & Opurmeeg TOO 7 12400 aan. ~ 01:00 pom ee ‘ALMOLOGY, OTOLARYNGOLOGY 4 RUZNOLOS sez 1s Which Of th bios © fo! hrombasigz” “Mowing is a mainstay in the treatment of cavernous sinus + Surge: Bs ey Antibiotic c. Heparin D. ‘Steroids 22. wnicn ch Of the £ Waich Of the fottowing mans the forte putetione will Nor ea x fopibestbisyanetant (Bgay pont WN) NOE cause 9 fe e cystoscopy exam (DRE) + thethrel Catheterization elevation of 73. In metastati letastatic rena: 3 cell cancer, the following conditions carries a poor prognosticating index BRCEDT + high serum lactate ° B. high serum calcium '¥Srowenase + high performanc i D. low nbexfgrmance (functionality) status score, 74. TRUE Statements regarding subdural hematoma are Is The hematom Ii: The noma eena at S*Pand along the inside of the skull. iy Stop at the dural reflections like tentorium Gerebelli and faix cevebri é wil cute, ‘ ill acutely show a hyper dense créscent-shaped lesion on CT scan. A: I, Ir ee ©. rt, 111 ot) rrr D. I, TIE 75s The ris: le risk factors for the development of subdural hematoma are __- I. very young and old patient II. cerebrospinal fluid leak III; anti-coagulant therapy A. I, III c. It, 11 Bs I, I D..I, II, IIT 76; Calcium stone formation in a patient with hyperparathyroidism presents with the following EXCEPT __. ‘A. hypophosphatemia Cc. reduced phosphaturia B. increased calciuria D. hypercalcemia 77. For patient undergoing adrenalectomy for hyperaldosteronism, in addition to hypertension, which of the following should be controlled pre-operatively? . A. hyperkalemia Cc. hypokalemia B. hypocalcemia D. hypercalcemia In the treatment of renal cell carcinoma of the kidney, radical nephrectomy means removal of the affected kidney and including the following structures EXCEPT __. A, ipsilateral regional retroperitoneal lymph nodes. B. ipsilateral adrenal gland. ! Gerota’s fascia. D. cer eralateral regional retroperitoneal lymph nodes ral hematoma is caused by a tear in __ venules ic epidu: C D. arteries 78: 79; Traumat. ° ‘a. bridging veins B, capillaries TODS Ver 2-1-9 Continued on Page 10 PaYSICrAN 14 Sunday AM EiSensure examination jer 20, 2020 - 11:00 a.m. ~ 01:00 wage 20 Ms ~ 01:00 p.m. oe . SURGERY ¢ OPHTHALMOLOGY, JARYNGOLOGY & RHINOLOG ser p 80. Which of of the follow: Prostaticie? ‘lowing is a FALSE statement regarding acute bacterial Br OS:AS caused by aacending colitorm pact: etenet oaegone! em bacteria. : : gochett iesetan® is managed by transurethral . + Prostatic ebaces Si Bfogt8tic sbacese requizes drainage via open perineal route. iknediatedysnttPictic should be started empirically 81. Which of the follow: eeefenes She, feitowing drugs 4s wor used in the medical management of ‘Ae Aninoglutethimige’ “Ieense?, B. Ketoconazole S: Metyrapone | 7 However, e2. 0 The following eye diseases present with an soyte red oY. a cozneat following poses no threat to vision? | Stbconjunceival C: Endophthalmitis remorrhage Di Acute angle glaucoma e right eye, 83: A patient consultit S,2ehstanadeatete' ego protuee porwient.atgonarge fron & A; Allergic conjunctivitis ” + Eerpes simplex virus infecti Cs. Gonococeal conjuncti 7 ‘Ds Blepharitis 3 wikis a4: In doing bariatric surgery, for Moe eaux ney gastric gery, laparoscopic approach is best, for __ B. vertical band gastroplasty ©. biliopancreatic diversion D. duodenal stitch 85; which of the following statements regarding inhalation injury is INACCURATE? ‘A. Inhalation injury together with skin burn injury will increase metabolic demand. Smoke inhalation is"generally benign as compared to skin burn injury. C. Smoke inhalation results from combustion products that cause direct mucosal injury. Smoke inhalation causes direct heat injury to the upper airways: Be D: 86, A 21 year old patient is diagnosed with Herpes simplex conjunctivitis. hich of the following drugs is contraindicated? R: Trifluridine eye drops B. Oral acyclovir C. Prednisone ophthalmic drops D. 3% acyclovir ophthalmic ointment g intravenous anesthetic drugs causes an increase 87. which of the followin Jn intracranial pressure? : A. Midazolam c. Retomiie B. Thiopental D. Morphine ivitis sicca can be exacerbated by __- eyes or keratoconjuncts can b _ 88. Dry eYeating tired eyes rubbing the eyes i fully blinking D. lubricating drops TQps ver 2+1- B, purpose: Continued on Page 11 PHYSICIAN 1; Sunday, Souicensure sxanination ex 20, . SURGERY ¢ oan 2020 = 11:00 asm. ~ 01:00 p.m. page 12 . HALMOLOGY, OTOLARYNGOLOGY & RHINOLOG SeTB 89. In a pati patient aralupstsent Presenting with to red ne Prementd an acute xed eye, emergency ophthalmic Ae history’ Nom ended Ane patient with mrss spazsency B. history of te { pAStory of id transplantation. D: 2-week aft ntivitis. ax aoculer sur: 90. After by inte ex bariatric sur: sKCEPr ric sur: +y, the followup should have the following gon2s + help patient adj: Ay? provitie senty eae isto easting patterns * Gc. Eoouide early id-~ ification of post op complications - D. recommend measur: eo ita ‘bariatric procedure. ij suri 5 to limit complications. 91. Gibbus deformity’ 4c . saa te aetomndty’ io’ a “form of structural kyphosis found in upper lumbar and lower thoracic vertebrae where one oF nore Ag jacent vertebrae become dued. This deformity is a result of B. leptospirosis’ c, tuberculosis 92 Ptospirosis D. osteoarthritis + Inguinal i ae Mapeee eet common in infancy and results from —_~ B, weakness of in C! Fonoral hernias et Cane? f200% D. patent processus vaginalis. 93. Virilizin: ig adrenal tumors cause virilization due to an adrenal hypersecretion of the following . E, Betradiol I; Dehydroepiandrosterone III. Androstenedione A. I, TIT c. II, 11t B: I, Ir D. I, 1, Wt 94s Common nerve palsy after hip arthroplasty includes the following nerves B; sciatic nerve . cc, tibial nerve B. femoral nerve D. peroneal nerve 95. Which of the following statements jis FALSE regarding cryptorchidism? sich echidepexy is the treatment of choice. 2: Ofeintreated, it is a risk factor for testicular cancer. C. cryptorchid testes are more ‘susceptible to trauma and torsio a D. Operation is indicated if testis remains undescended by 9 years of age- ; 96. Patellofemoral pain syndrome, alse, knows as runner's knee is an anterior Patel tein involving the patella end —- S. Bnterior cruciate ligament B. intraarticular compartment: Cc, medial meniscus Dp, retinaculum 97, Blindness is @ consequence of the following conditions __+ corneal vilcers ie qx. glaucoma Irl- uveitis ee tp kt BR. I, IT Lut Be Tr aI, 11t p. I, IIT continued on Page 12 mops ver 2+] PAYSTCTAN 14, Sunday Censur, xamination eo sues a3 SURGERY & CruTuanwouocy, OTOLARYNGOLOGY & RIITNOLOG 98. Which Of the following statements is TRUE for omphalocoele? 3 Remnants of amnion are reabsorbed 99. Compartment syn: ‘an occur in any extremity following trauma. If recommended for the following conditions ‘EXCEPT __+ B. ischemic period of more than 6 hours. D. combined arterial and venous injuries. ‘ 100. Which of the following is a cause of acute painful visu an associated red eye! A Retinal detachment cy Pituitary apoplexy, Bs Orbital cellulites D. Ocular vascular ef see eNO plete) sot will : tions (Com| WARNING: Failure to submit your Test Questions (Complete Oe cause the cancellation of your Test-Results Negi ic at PROFESSIONAL WEROLAT H Hanthe ROKRD OF MEDICINE PUTSICIANS Licensure cxaminalion Monday, Harch 9%, 2020 ENTETOLOGT pio weme = $0009 Aare. INSTRUCTION: correct anaume fore tollewing quertions, mark only one, ge fot enn ttwm by aliating the bow Sor tespon Tete TSE your chinten an EPRICTLT MO ENASUNER ALLAWKD., MULTIFLE CHOICE agaistes 1. wnat complenant disorder causes huredLtary angtowcens ou activation of Kallikrein? Re ci estarane Inhibitor detieLency BL CS-C8 defictentien C1 Csb-9 daticlency Bl C3 deficiency eo dovnragulate in the wberve? 2. What does progeaterc A. Facaptor for ACG B. te own receptor C. receptor for beta? adrenergic BD. receptor for luteinizing hormone Which Hunan Leukocyte Antigen wubeype i involved WEEE multiple scleronip ond SLE? a, D2? eA) a, DR? Db. DRE 4. Renal Blood flow in human |e 2 A, 25 percent of Cardiac output 3. io percent of Extracallular flutd C. 30 percent af Cardiac outpmt Bl 70 pareant of Talal Body Water leulnr preasure-voluma Loop? . ALL of the chatees 5, What factor affects tha ventr D, Increased afterload aj prelond ed Contractiiity 6. what type of contraction ia when the force a than that genuine to ILE oF mova on ols }eou? ‘A. Taomatr ic c. fsoronle B. Hanotonle D, Trokinatic attachmant of ba 7, What Immunoglobulin prevanta peabrane, and does not FLX cumplemantt a. 19D c. Ty Dp, Igo B. Igh What are the functions of muscles? A, Thermogenanls B. constriction of organs and 4 ft. ALL of the choice D. Respiration . Continued on vege 2 Toes Var 2.1. RORESSIMIS Leama = ino am, euYSTOLOGT Following Works ly the 1e1 achantem for hormone action, ©. Corthaol bo uayootn othe hearey er than 70) parcent votha het thal ar tian 6 Following muvemanta rerpiten qreater Hunter af muscle 1d Paated mvemmnta C. Larger aowmants. Finer mavem DL Cnatnne movant 12, nth drug thm duration of RRH? » Bantodiasepin ©, bhanseyha eam ie 8. Alea? DL Anphe tom at the able to per 12, Which aEst describes the physlolagte nat aac 8. Approximately aquivelent to the anatomic dead apace in normal lunge c. Bota Tu D. Bath are Falee , M4. A 4S yuar-old man was diagnosed with uretaral stonm, what would you expect to happen go hfs canal plasma flow? A. Oscrease &. Ho change 8. Increasa ©, Variable 45. Which eccurs during the Late stags of exert A. Cardiac output ie maintalned by increase Haart rate only A. Diastole im prefarentially shortenw! with decteased heart rate €. Cardiac output 4m decrensed because of lesz filling tine D. Cardiac output is malntained by increase Heart rate and stroke volume 16. Which lung volume is composed of the Tidal volume plus inspiratory reserve volume snd expiratory reserve volume A. Inspiratory Capacity B. Total Lung Capacity C. vital capacity DB, Functional Residual Capacity 17. Tyremine rich foods can increase levels of Honoamine oxidase and cou! which elintest problem? A. Nypergiycemsn C. Hypertensive crisis A. Hypocalcamia D. Thyrotoxicowis 18. Which 1m the procine site of fertilization in humana? A. Intramural part of the ralleplan tube a. Ampulle of the Fallopian tube Cc. Isthmus of the Fallopian tube DB. Infundibulom of the Fallopian tubs Continued on Page 7 ‘TODS Ver 2.1.9 FAYEICLANA stowneure examination SOUSICIANA Klowreure examina les ip, = 10108 4 ‘HERIOLOGY - 1S) Whak Le calted . Re Titeric Marviue fyeten Bl Autonnmte Naewntia ayel em C. Wypothalante Fitnitary ARE Dl PATamyRpAt han ic Macuena fyeter Liary vedge preemure Naber 20, Which Inetvunant mansre (he poine AL Cyr Line cm 1. Elect cocard Logeam ni ater al weanan of charge? lex Ps Which rot tes aaniute Ay Subdiminad occ tials B. Flexor withdrawal raf lee ©. Golgi tendon cefle D, Stretch reflow Lute permeates 2 secbranel Hrumion coaretcte ntiecrion caw! fies 22. Much dencribes the A. Partition eset tle: 8. @amolarity whe whLen s 22. Which 18 the functiolt of complement CibT n. opsonization ee chmmocaa ie B. Anaphy lexis BD. Apoplowta . 24, Whiten endoceina complication, cai aries fram acromegaly? Ay Glucose tntolsrance C, Arthritis B. Weare ferluce fi, Hype nyroidian red= to hrestne Ln whet way? 25. Patients with asthma “1 A. ALL of the chalow B. Breathe in short breaths to compensate for the high airway realntance co her Lond toLumn D, Mreathe raplelly 26, The following in true in cases of diarrhea? A, Hematocrit decronnas B. Plasma protain concenuratlon decreason C. fer valume dacreasas but no change occurs in the ef ECF , wpce will not sbeink osmolarity 27, Which of the following stinulates peripheral chemoreceptors of the heart? A. Doctease PCO? ¢. Increana pag the blood fi. Decrease pli of the blood 1, Incraama Po? 28. Which WL1L NoT activate the chemoreceptor trigger tone 7 A. Emation Cc. Patty food B. Veetibular stimulntion be Radtation atrythroblantonta’ fatalla dua to mh incompatibility? Juce anti- D Tg which can erase the placenta B. Mother can make anti-D IgG that con cross the placente ¢. Mother can produce anti-D Iq which can cross the/placenta D. Mother can make anti © entibodies that crosm the placenta ueches than skeletal muscles’ mic reticulum 29, What cau A. Mother an pi 30. Which structure te more common in cardiac mi A. Mitochondria Cc. eadopt yuclear Inclusion u, Tubules Continued on Fega 4 Taps Ver 2. inet = toNgd meme Faye ‘Oaioo mame fers syuthaste of which slotting “a Ytanin & datictency e* Ae Factor Vi NN. Factor rad i copmulraltona Ulth hae pkyatetan foc Ero slyapapata, bho oan « frequent rine Leumi tatad with. foul tatans, ee ee (ipa cabommeandat ton ak hat Fee eee tagnoat iim ani! micwhomieaL Casta turned out Ghkt te the Ltkaty comdLiboa of the pact Re AchLorysthe Ni Peptic ulome dt Cl hnwlety disorder DB, Funetional Ch trointestinal ab preal fic juice by the 32, ALL of the following wATL tee stomach’ n glands EXCKET ubichy A. Texture af Pood smell of real a of Fout b, Sight af Food ce Test 34. which substance can inhibit 7 A. Caffeine oride solution 2 Aleohot wnat ie the partod during witoh another setion potential eannot be Siiclted , au mattar how Lerge the #t{muluey A. Exhalistion B. ratigue Cl Absolute refractory period D. Relative refractory pariod ine diwense? hoella Liniag the filtration 36. What causes proteinuria in glomeri Lion of the endatiwl ial fad tnereaserd uncot le pressure of th capillary Lining Edema Fae renoval of tim antanic barrier Linking the Filtration barrier Oe eee et the basement membrana of the endothalial sells 3} of tha cardiac potantial ? 37. What occura during Phas wcronses, K+ incransae and therefore in. ca? + conductance predominates a, als conductance Lnereases, Ke doeronson . Te cauned by 9 transient tnereasa in Cazy aonductance which reault in an inward Caz+ eurrent D. Transient inceaase in Nas 38. Which 4s WOT # factor fac nN € tha heart? A. Adenoalne B. Hypoxia 39, Which refers to the shift Ln oxygan Ehanges in the concentration of earban it nvieonnant? oe Bs cc, Naldene effect A. Bohr Effect B Uibba-donnen Equitibriun p, chhoride anit tivsoclation curyagcaused by cn dioxide or thé'pe of the Continued on vega 9 gos Ver 2.1 FRB CHAE ‘om enneurs aay, March a5" rm TALOEgSY sna thon OO asm. = 10800 wom Pages a0, white AER be the dirsetian of blood thow starting from the Pulmonary vain, Right Abe si v mL ventelole » descending Pulmonary vein, Vell a! tle valve, sock, €. Pulmonary acto, ” Aart le regivgtentt neractile tissue Of Lhe GE track is almost « ect AL Striated muscle Cy Clrcular mest BL Longitudinal mincle By Unitary omeoth 42. which in the alte of secretion of intrinsic tector? A. Gastric antrum Cy Tleun Gastric fundue By Daodanur 44. Wich dn a correct statement about the vartationy of ot and Li levels ever the Life npan of both miles and feanles? All af the chatcen Ae childhaed hormone levela are lowser and Tul > bi In pengacence hermne levals aro lowest, and PeieLll At puberty and reprodietive years FSH > bil Lat which action potential dn ineyitasle? 1s the mombrane paten Threshold Depolarization Relative refractory parkod Ang membrane glycogenolyais in the hepatic celle? Glucagon Dipeptidyl 45, Which of tha following atimulate Ae Amy a, Ingulin h 47, wnat are intercellular connections which form attachnents eften Darwech epithelial calla? ‘A. Zonula occludans 8. Network €. Coupling cells D. Gap junction 40. What are tho functions o€ the bloo#-brain barr loc? A. Tt reguiata tenperature of the boty to adjunt to environment B. It nervy ahion of the carabral cortex ci Tk server Fawervair for drug meLabol tt BL Te prevants the anceps of neurotrananitters from their functional altea in the CH! eribes a inotor unkt EXcErt whieh? ingia moter neuron and tha musche f4bere that 49. The following a ‘A. consiate of it innervate: For fine control, 6 single motor neuro! few muscle fiber few Darger movements, a single motor neuron may inne thousands of a ftibere p. Conaista of sevaral motor neurone Apnecvoting many muscle fibers Continued an Pago § TODS Ver 2.1 B in innervate’ only @ rvate c. MHEICTANS Licensure paam{ nation cancay, Harch %, 30370 = ortll oe bOeDO Rome Page 4 wsroogy Ls 30, Hi fy ant two point dbacrinknation at , | eke et Pine tavet ant v jLacr int a n. athuny 7 unnayae D. Anterolatm vtatmeal 51. The following te true about ouutacion and [nchatton EXCEPT whichy Re ovulation famt partum ter ) saineh even withow thy bee rat Pratmecin a Tareattean (nniiitt Traimccdy au tig tacrat ton ant eat l fin at bib meet FSi on the ovaries 9, Pots fine thin action of i antecter Brew re UTR y anninits Psi ml Lil secretions 57. wat happens Lf you cut the qeniculocaicarine eract? 2 (etkptawten emnntar aper ke MUtatntereL hemi ancple c. mianepim DL Metinal bntachment 53. Myogenic mechanism of renal blood flow ts — Sore UnIch increased fanal arturial preawure Peads to incre Gelivery of fhul: Ve denne ®. accempliahed by nal vasculag Fesiktonce by hanging poxicion Cc. Sceosplivhed through atimulation af the aycnerharts f tystem oo rENich une renal afferent arterioles contract in Feaponse inyhich Lhe raaintain eametant reed Wud flow $4, which te releanad fram atrial myocytes in remponss te ineressed biose volume and atrial pressure? A. Aor bic chomorecepLors B. Atrial chemoreceptors CL atrial inatcople peptide D. Atrial Matciuret Li peptide 55. wnat dows the pulmonary wadye pressuce clinically Measure? A. beft ventrigular pressure RL Pulmonary artery diastolic prussite C. eft atrial pressure b, Pulmonary artery systolle pressure 56. wnich of the Firat branching of the branchial tree has gas exchange capmbitiet A. Terminal brooch late Cc. Sagmental brencht f. Respiratary bronchiale: DB. Meium size broncht 57. Which cell in the distal tupulo are nanwitive to the concentration of nodium chloride 7 A. Padacyte c. Hesangial colle A, Macula den: fn. guxtaglonarular cell se, Nuscarinic receptare causes the following EXC T whieh? ad gaairic motility AR fo Increased secretion of the alanis &. Jed conduction velocity bn AV nade o. ed heart rate continued on Page 7 ‘TQDS Ver 2-1, wage 7 a BYYSICTANE Gieenaure Exanint ys Mare o, aoa = om ive ron = 10.00 ame 59. What Lam marker for PCT? ce. mann Re kn bey ear rn RL Pusne’ ius fy Tritheced Waxer orpibon? foterem} lular ¢pace of tvommotic fhu 60. wn Site colle ss y bacon co ueoet ateral Intercallular apace + bleed c. T, pacienbular eaptitery blood, emiLutar WpRe® Cera ey ieeet culate mecd, pacitinutae oaplbtery pieod, Lataral tntercel kular S9ne® 61. A 16 y a bay wma and homing be er of a dance group. He a UeY Bet ing the apim, Nin eye snap quickty Co the lett. sping tot The Ener oye moivenent | A. Ataxia B. Piplopla G2, wnat pappana to the mann volves for arterial POF and F502 caring exerciae? A. Bo not change ¢. pacrasen a, increase Dy Variable changes artes by PSH and LIT c. Owlation D. All of the enolces ventricular tibritistion withost CFF ond 63, Which are atimutated tn the A. Follicular development: B, Lutelnization 64. wnat is a clinical sequela of dat the (iiation? A. Syncopaa BL Congastive heart £aitu C. Dyspnes and cardiac D. Fatal arrhythmia 65. Which of the following te davolved i N pontine reticulospinal tract BL Tectopsinal tract ¢) Rubrospinal’ tract BL medullacy coticulosplnal track G6. whieh form af the syqate Ingtanbe {ie the maternal ¢. 0 cell eg9 ches fn the control of neck uaches? uterine wall? A, blastocyst 8, morule Dy embryo 67. Whet Ls NOT & major anion of the ICr? A, Proteins c, Adenonine diphoaphare 1B, nco3- D. Adanosina triphowphate hanct Located? Cc. Vonkriquiar muscle fb, Av node gg. wnere are the ceceptore In tI A. All of the chyloes a. SA node §9. which of the following Block retaane of Acatyicholine fren presynaptic terminala? h. Hemicholinium cc. Neoatignine B, Batulinua toxins Dp. curere TQDS Ver 2 continued on Page 8 PAYSICIANE Lis honas censure Raominntion oan rage & ay, March 3; nage, — onto eee .ynina ae ‘WYSTOLOGE per A ow of hye 170, Mich contractions in the GIT © movenant? A, Migratory DL Mixing and grinding Ce Rhythmic pianbe cer 1, Seymentarion 71. which cyteninen media cachents tn nabianancy? A. THT D, th? ge. wnat with be secentell 310! purmotactan of th CAFELRE colle? trevor ee MeoI- and ines ineys Me Wel and iacrinste f or C. HEL and WOOD pb. He ni Qa. Mich te HOT) tL eRe be almaat alway present D. te conaiets primgrity of bne PMUeP Te comet idytenering (Orre co tbe doticdeney OF ‘absence tts eta ayricome 11) peoen p. it eeduces mit face kension dinrupeiog the qatermaleee molechte By 1 wanke de Of gast jotipid dipalele pannatal Eeeparee Mee he alveols cable Ut Patuenn Liquid Th, which embryonic StevECurs develops inta the @Ale genital erat? Ro urachis OF fariLerken Socts Al waif fian duete D, vittebine 4ct 35. In the exemple Shoe) grat is the direotton of the uree fluxT A, High to bow Tancanteacton Bt mann of the eholces Tn equi Live bar Low to high concantrat lon je an antaganist of ene 2 receptors? ‘i yohimbine . Phenoxybanzamine p, Prazocin Hatoprolal t pan clearance | in qwanoringsgenal Blood tlov? 77, what is the Limitation of wat FE overestimates f teed Chev by LOY BL fe can be used FO angeure ranal bloot OF ow of abiperts of et the kidneys E c. Pal is secreted pur not filtered PY ail par! 2 kidneys p, Te undorastinater erne RAF by 108 i " or qe. whieh meen to the adenylate oye) mechanian Of wae actlon results verge action of Norman z a A. GOP release trom & proven Be Rettvation of theo repratn af adenylate oye Lesa n Rinsse & c. converslon of ADP ta ATP ins by PCO which synthes 170! the hormone of the posterior pia yn. Secretory vesicles from the lymphat be of the prain B, secretory granules from the c, nerve cell podies in the hypothabant Kvecory granules comin trom the 13. continued on Page rans Ver ua as. O65 87. an Cee ew Examine it = 01 OG mime Page 4 hareh 8.7030, aes ke the won on tLe the wont come BL wane of girth of a wusete7 hi Wwnten of the te mitagn te th Amylase wopet 2 the common aie OF apanta Uheumer horas in the hunger A. Aper tc. Might bang : Bo hha tung 0. Wathen at the Tange Following ta HoT an action of tow A, Epiphymwal closure 1D. Deapening of voice C. Pomttive feadhack on antario: pituitary O. Elpida teronel Given: Solution X {# 100 Wt RCL, and sulutlon ¥ in aM FCO What Vill happes LE Saluttons X and Y ate separated Ly a reaante nenbrane that ig pemnmaile to fe, but wit to Cl? Re will diffuse Crom aolutlon ® to melutton ¥ until s Potential develops with Salut lon Xk nogative With reapect v A. Ke lone will diffuse fram Satulion 7 tu golution #6 the [RF] of both solution ta 5a.5 att BL re WLLL Alf fuse trom olution x ta Aolueion ¥ until = momhrane potential dovwlopa with Solarian X negative with raspent to folutian ¥ Che done will diffuse from Solutinn & Lo kelution T unt IR+] o£ both solutions 4s 50,5 mH D. RCL will diffuse Crom solution X te solution ¥ until the [RCL] of both solutions Ls 50.SmH 1 Choose the correct sequence af action potential conduction alesg the Conductive tissue of the haart. Ay AV node->8A poda-onundle of Hin->Bundle branchas->Pukinje tibere 8, SR node->AV noda->Purkinje fiber-sundie of His-»Dundls branches C. SA nore-2AU node-rBundle of Wis~-Hundle branches->Purkinjs fibers fb. fA nade-> Bundle of tLs-sfundle branchas-2AaV Node~>Purkinje Fiber Which blood teat can be dona to measure activities of the extrinsic pathway of coagulation? A, leading tine . Activeted Partlel Thrombopinstin time C. Prothrombin time O, INR Whlen photoreceptor is rasponaibie for aight viston? A. Opadn + Bipolar calle 8, Rode fo, Cones Which are actions of dllydrotestosterane? male heir pattern te ho mitvferentietion of epididymla, vax deferena and warinel vesicles c. growth of prostrate D, male pattern baldni Continued on Fage 10 TQDS Ver 2 te LoRGICIANE Licenaure taqminet inn che Vege am, — WNIT eam Page 10 | WetoLocy ers 1”. MiiicH 46 canponmibte toe une anima men ot MER tn the Gite As Embryos OE the prodimal nal ne Do portsee saicinne! ation canicact ion Be MD action atthe erreniac and longitudinal sanclas of the $8. Which nerve in aftectad ann yatsens wmge Qalvta Ci lbe becaiee Wet ghe ering Lea Tce tan Stigimait of pelvis throigh hip abasetion? bas ee rarbat io nace RN. Cbturator nerve 1, Common peroneal eukve WL, What La the effective eafractory parted 7 Ae Te be the period during whieh al ection potentink can be elicited B. 1t begins with the upat After the plateat C. It La the period ducing which 4 conti can not be elicited DB. Tt da the parlod inmaiiately after the mation refractory pariod uhen Teplacermnt ia alrout eimpiste eat une tian potential and ends tal tion action pote 92. What fo geatatinn da the alvaular dunts become tessinal , a te birth to first 2 weeks of nunnatel ©39 a. to tirat 2 veek neanaal ays c. te pirkh D. 70 wemkn to birth 93, Which druge bieck voltage wensitive Wa channels and abaliah action potentinla? A. Lidocair cc. oigltelin B. Arsenic OL guehain 4, The following events oocur on {Inhalation EXCRT which? A. The diapheagm descends cee eatminal museles contenet and push the abvominal viseara ceanial ly c. Tho external intercostal muscles contract D. The ribs are raised 95. Which condition t# peliaved to be dus to the degeneration of the dopaminergic neurons 7 A. Parkinaon's disease c. Schizaphrants Bh. Hyesthentas gravis D. Alzheimer What substance couples hormone receptors to adjacent effector molecules? 'h. Adenosine triphosphate f. Adenosine diphanphate B peeranine keiphouphate binding protetns D. 1.4.3 teiphonphate ef 97. Glucose teanspert in emmcies and adipose cols is an exanpl fh, carrier-mdiated Lransport Bb. Facilitated diffualon Cl Primary sctive transport D. Simple diffusion continued on Page tt Tops Ver 2+ PUYSICi ANS 1, Monica icansurs naaminat (ot Fee ee Eat an aim, = 10100 mam rage 11 HTS oLocy in cay SITUATIONAL, ere asthmatic wttack. fe experiences 1 o¥id tm tO mm hig ana the following statamant about this patient Le owt Likely to be © than formal pavause of tuadequace arterial poz J Gnn exchange Q, forced mapirataty volutial for is Lncrussed So Vent ilerion/pertision ratia be ber arcas of his lungs D. Wig arterial PCO2 16 Lower than nnraal bacaiee hyposenta Le causing him (o vdneiiate igh vital anpmetey ( FEVIFEUCy Wl Ln the affected 99. To trent the patiant, physicinn should oduintater which ef the folioving? Ay Any adrenergic antagoilet B. Sete | adrenergic agonlet ©. Bete 2 adrenergle egontar DB. Muscarinic agent itive to this stimulus 100. Central chemorecastoca Ly Lhe madubla are 5 thet ineranse breathing cate ? A. Low COR . C, Wgh en? Low pit bp. Magh pl see eno ee WARNING: Fallure to aubmit your Taat Questions (Complete) set will cause the cancellation of your ‘Tast-Results forthe subject. Saat lio.t public of the pburipeln PROVESSTONAL REGULATION COMMTSSION nanile OAR OY ENTE IIE PHYSICIANS Licennure Examination Sunday, March @, 2020 02100 pene = 04100 p.m. . ct anuvor far wach of the following ark oniyona_ anaver for ench item by shading the box Yona RURMET Te choles on the snexer sheet provided. SS N.LOHEO. questions corresponding to STRICTLY NO ERASUF MULTIPLE CHOICE Ly what will be tho HOST PROBABLE consequence far 6 tenale with pelvic In€lanmatory diverse from gonorrhea? A, Uterine dngenapation G. Inferuility NL endometrial CA 1D. Gloneswtonophe tt ia Lowhat La the major kargat call for Epstein-Barr virus (EBV)T 2 Wacraphages: lear enll (smth) 3. Which antdbody tivers indicate recent or recurrent streptococcal Infection and are found wore often in rheumatic Individuals? AL Anti-Dtame 0 C. hitL-Streptolyxin O BL Antichya Lurbo dase DL ANti-H protein 4. Which of the follawing NODT APTLY describes the disease process of Dipther tae? PinGnvelvenent of the hart, kidney, adrenals and other organs results from military spread . By The clinical manifestations result, from the inflammatory response to tha organtsn. c. Ther clostridium diphtheria tavades deop tlasuo and enter the blood stream. " b. The clostridium produce toxin that cen be absorbed and result in distant toxic damage. 5. nelde from facal swear, which of the following mathods say be used to discover, view and diagnose Whipworm infection? A. Scotch tape method A, Iodine formaldehyde tachnique Ci Saturated brine flotation tachniqua BD, Potato agar G. mich of the following poauensen diract cytotoxic ectivity with rhe ability to kL11 {nfocted colls? K, Cytokines c. B= Lymphocytes B, COBT - Lymphocytes D, cod T - Lymphocytes J, Which of the following i primary target coll of IVT A. Plagma colls €, Potynorphonuckear celle B. 8 lymphocytat D. T lymphocyte Ay otheruise normal healthy 35-years old nale suddenly developed profusc Miarrhea, acconpaniod By nausea and voniting. ‘stool axam showad Gramenegetive bacilli and biochenteal Teection pattern showed thet Grgoniom is non-lectose fermenter, The diarrhea regpanded to Loperamide orgepacient was well after 2 days, Nich of the following was the Continued on Page ? TODS Ver 2+ PRYSICIANS Liconsure tamination Sunday, Hach Bs 2020 02100 p.m. - 04100 pos Pago ? ALCRODIOLOGY AND PaRAssTOLOSY Probable of fanding argantan? 10. 11, Wnich of the following & pathogen In Alps? 12, A 45-year old man con Nin left kane. On examinatl overlying kin was nat warm te touch and non indicating prenonee of fLutd. yellowish non of the fluids examination Lo tdentify the organian Invalved vould be + A nagative-nense HUA viruH has the folloving Mistinctive property, which What ia tha HOST PREVALENT oF DactIlus cor Closter lillum partriagens A. Stophylacoceum av cule B. Streptococcus viridlann ane? A. Ite molecule functions an ay ehh within the Infected coll, B, The vielona carry an HUA polynorann, €. The ganome in single mtr. 9 D. The Lsalated HRA is Infockboun, min infected cat bites? ° c. Gaphylococcus auraue DI Straptoceccue viradlans A. Dacteroiden BL Pasteurella multocida using brain abacess in an important A. Candida B. Coceidiodotodes tmmitia €. Aspergillosis ap. DB! Ceyptococcosla neofurmaun Hted at the OPD for sudden painful swelling of n, tha left knee van suotler but the nder; (+) balloumont Xeray vas formal. Jdce of clear ‘aod microscopic examination st useful dingnostle tb Elud vam aapirater we wo MIC Or RBC. The &. Tuberculin teat 3B. Gram stain * C. Polymeras# chain rvaction Dy Culture 13, What ds the major complication of Palvic tiflermatory Disease (PID|? 14, Which of the followiny will MOST LIKELY cat are Fond of eating dog mast? 15. Which of th A. dynmenorrhes B. dysfunctional bleeding C. infertility 0. recurrent abdominal patna disease among thosu who ‘asclola hepatica A. Onehoeeres volvulus c. chinococcus granulosus AL Hymenolapaia nana B following bent defines transposons? A. Hobile ond transferrable segments of bacterial DNA 8. Proteins expressed by the chronosomal gonew of the bacterie C. Viruger that have infected a bactorta DB. Enzymatic products of bacterial genatic expression 16. [ts rainy seacon again. From the newspapers, Tv and radio, you will haar the advice of cleaning up water-holding recapteclas and plants containers clone to aur hones Why 16 this #0? A. Hoaquito breeds in domastic water-halding receptacles and con vanity travel to hones and buildings Stagnant water becomes foul smelling and murky, good Braading places for ineecte. C. Plants will dle easily in dirty receptacles, D. Molst plants are hones to mosquitoes. ,which can travel ta nearby hones and schools. ily Continued on Page 3 TODS Ver 2.1. PAYSICTANG tLe: Sunday, March @ ure Examination 2020 82100 pam, ~ 0400 Pome Pago 3 MICROBIOLOGY AND PARASITOLOGY + 17. Which of the fotiowing In a MAJOR virulenae factor of streptococcus 19. 20. 2. 22. 23. 2h Prodenon? +H protein DLT Substance Cc. Steaptolysin O 2 pravioumly apparently heathy yu might to tha ener gency ra tvagaterat wasvonii with wynseeical Seer axtianities, Thin wea preceded by viiticnity wt paatiowng, end reat van atebrlivs th Uie emnegancy room, he fsddenty wont! int weap went ony ALTE and wunt inte irreversible aera eeeets gpon Interview With 9 colativa, LL vas revealed thet he cee ietover canned wardiinen for lunch waveral hours ago. What te'the post probable dingnonin? A. Rotuliem A, Nisgeria moningltidle peer an Tha at vied C. biatertonis Db. Totanun fC the folloving divtinguishex adaptive immunity from innate Wate o dnmuntey? R. Fro-tnflonnatory aytokines B. Antigon-upecitic antibody . Phagocytom Di Microbial nensore . What is the HAIN Immunoglobulin eeaponsible for mucosal immunity and protects micous menhrane from attacks by bacteria and viruses? A. TaG e. to B. IgE ‘ DL TGR Which of the following infecte human by penetration of the lary through tho skin? A Necatur Anacicanus c. Meania Soltua 8. Trichurds trichurla D. Aveariu Lunbricoldes hat is the HOST common viral infection of the reproductive system? Ae Malluscun contagiosun N. Genitel herpes CL Auman papilloma virus DL Hunan Inmunodeficiency Virus wnich of the following atatenents ia TRUE regarcing poststreptococca? diss 2 Re ettistreptocaccel chanoprophyloxis is recommended tn giomarulonephritis patients. a. Zlther akin infection or pharyngitis precedes rhoumatic faver. c. Rhounstic fever has 9 marked tendency to De rooetivated by recurrent streptococcal Infection. Oe eeeee Se nepheltia and. rheinacio fever acaipimttar following untreated related infection, t ts in which of the following dimaseas Le the chon Le fon ‘foynd? ia A ye primary tubercula: B. Secondary syphilis ¢. Leptospirosis D, Cytomegalic viral disease Tops ver 2.1 continued on Page 4 DHYSICIANS Licensure nx nation Sunday, March #8, 2020. - 02190 pms ~ 04100 Wem 4zcnos to} 25. Suddan profuse dlarrhas scconpan! a water= lon iebalance tn Ne a. P_PARASITOLOG! hy vomited stent, viich can bem By what organian?: : Ce Bitgod Viteio cho Salmonetin SITUATIONAL Situation when but recurred at during the febr Ue yoctod vagy wan 3. neieimgan bey flaccid paralysia and moact alropiy. 26. Which microbial agent sow the HOST pr dl at residual of the disea AL a. c. D 28. what a B. D. Situation associated with Low-grade afternoon fever revealed an emaciated patient with herah brn: Qugcultation of the lungs. No a or palpated. ne? Herpes simplex Mycobacterium ce. tatiovl in the pathogente i the pet tant? Inflammation of the mischos pagenmration of cho per ipl al Dastruction of the lover motor Tetmenent blockade of nanroteanie waives ie the goal of the Ht camp Prompt Ulagnogis and trentinent Global eradicatiun Reversal of the whole disease process Appropriate isolation strategy of patients © ald kady had dry coughs Phynical examinations ath sounds on nlargod Lymph pode vas observed X-ray nhowed inecaaned dengity at both upper Lung 2 = For the past fou days, a 40-yea fields. 29. Which form of tuberciilosts in the pat. a B, c. B. 30. What a1. In which layer of a tubercde ( TH granu Primary tuberculosis Reactivation tuberculasin Ghon complex Latent tuberculosis doops a ponitive tubercuiin toat imply? Nonence of primary Infection Immunity to the dine Infection (n the pan Active dinease and cultured? A B. c. De Central zone peripheral zone of Fibcoblowts] ynphocytoe Hid-zone of pale epitheloid celle Peripheral fibtoue tinue conkioued on Page bl Glenteiadium bovul Laue fb. terhertahia colt Ittere et the «ynapse gn in ardress thin disease? Jont manifesting? 5 to rapid dehydration and electrolyte ace old nan waiters from tha wffect of a childhood disease i by nial qrode tevas which subnided by muvee Thea thon. he tind patchy ar Lie rauie of ble ehildhood shantan which exploins the sequelae and Loma) can TH bacilli bs isolated mops ver 2.1. PHYSICIANS Licens = ure Examination Sunday, Harch B, 2020, + 02100 p.m, ~ 04400 [ome Page 5 sera HICROPLOLOGY AND PARASITOLOGY situation 3 - A 29-yeare at and sore throat, And gpl enor aka tine ph Slovated white many of whieh w trepvered Cully teoatnent The atten mononueleoste. developed fever, malaise, headache, vhinmination revealed’ lysphadanopathy vi axaminat ion showed elevated SGOT, SGPT, SiteetdIadirort Bilirubin. CHC showed nminance at LyRphocy Lem, Rtynteal telymphacytne, Parkent, e etake of conbinod honpital and howe tng phyticlanta diagnoats wan infectious 24g Wi the ettolayic agent of the disessa? 32. Which af the following La considers: thins herpes virus & A. Tpatein-Rare viru c, B. Human herpes virus 7 33. Which of the following prapartias tw the halinark tor thin virus? A. Transient aplania 1 Gell tmmorta tization Infection with C. Mawsive cytolysis 1D. Malignant transformation 34. Which of the following can alno bo caused by this virus? A. Burkitt Lymphoma C. Xaposis’ Sarcoma BL Hepatoma D. Congenital ononaiies ele and bone pain, # 14-year old boy Giagnosed Situation & - Aftor about of fever, m developed bleading, shock and eventually died. He w to have had Hemorrhagic Dengue. etont developed severe dengue? thon praviously. the same serotype am tho 35. What could be the ‘reason why thin pa! ‘A. Be had completed dengua vaceina B. He had a secondary dongue with cimary c. He had with a different sarotypa 99 previous. 0, fe had conourran’ wcondary dengit tt dengue and malorial infection, o of Penqua Hemorrhagic Fever? - 36. What is the XEY pathologic feat A. Cytotoxiclty against erythrocytes 8. Diminished platolot count C. Increased vascular permenbi lity D. Depletion of elarting factore 37. Whot tn connidarad the HOST rellable method to identify an active Dengue infection? A. Serologic dingnonss C. Tnolation of the virus Db. Platelet count A. Analysin of paired nora rc consulted the clinic bacausa of {oles on his right ng i-4x sinca 3 Situation 5 - A 32-yaar old male office worke: the appearance of multiple painful grouped v4 suprapuble area. This problem had heen recur year aqo. Host Likaly have? Infectious disease dit thie patie Folliculitia cc. Seals las Herpos proganitalin D. Syphitia 39, What Ls the HOST conan mode of trennnlosion of this disease? A. Through pediculowls puble lice DB, Sexually transmitted €. Thru inaecta? ant bit 5, prolenged contact with moldy modat eres Continued on Page & TODS Ver 2.1.9 PHYSICIANS Licensure xa Sunday, March 6, 2020. - ton 2100 peme = 04100 prams Page & 8 facRonroLosy A PARAS 13 40. Which of the following Laboratory proceduras 6e Taanch a ie row nba uaed to don AL the Situation € — A nuber of aduit patient in @ carter community neepteal Manifectad with aimitar clinical symptoms, The 411 starts With malaia, Fever, headaches thy nan -penduct ive Cough, Later, cough becane ALU aputam, womet tm Ca eranted. Alou! eraminntian shoved mils Imukocytosts the Mhowent pilmanary consol idat io Rial appeared phynteally wally cophalusportn, and vancomycin tailed Werythcunyeln ware effective, Hhile chest radiagenphs patients were up and abouty Treatment with panierit fatmant with Catracyeline a AA. The vaual bactertulogic method Tailed te twolare the organtesy Hnbch of The Yet iowing ta the HOST likely pactectal agent? ‘A. Haemophiqus int c. Stroptacoceys pnwusant Hlanydin proumonia b, Mycoplasma pheunonie a2. wnat an the plausible censon for thecopeutte fatlure wath Penicillin eee ye and success with Tetracycling, et, al.7 Ae Plasmin mediated resistance to penicillin, ot. B. Reta-lactamase production CL Bacteria is call wall defective D. Chromosomal yoneration of resist: al protetn 49. Wow is the diagnosis usually made in this tLinese? A. Gram-stain Dl tucleie nid amptification test C1 Clinkeal recognition D. Serology Situation 7 - Following worldwide campatun to erodicece, smallpox in 1967, the fon? can declarad wlininated 10 1780. llowaver, since the disgeee lon wan stapped, thore born Later are not protected against Lt. 44. Ana hypothetical case, where a S-year old voy vas DEONgne tO children" Medical Center because of homogenous vesicular eruptions most: sbundant fon the face and leas on the trunk and oxtremities, which of the Be eee Maduran would mOHL speci ticelly conflem thee the BALsehe has smallpox? A. Antibody annay B. Virus isolation by 1 embryo Cy Direct examination unier electron microrcopy Cr Takunohiatochenistry af viral antigens in timaul a/materials trom nkin lealong noculation of vesteular Eluid inte chick 45, Mich of the following wtatomonts rogarding smallpox 4s TRUE? A. Majority of caso of amallpox ware hot olinteally apparent (aubelinical) a. (a Rhtection gave compluta protect Lon against reinfection C. The Infect {ous virus had severe! seroLyrey C. The virus had a broad range of animal host aulde from human y the vaccine used against 46, which of the folloving aptly describe 1 «x? amOLIPOX! che name coupon views unas wy pavers Tanne tn 79 Bi re de a distinct specius of tha poxvirys ‘ C, re is perfectly safe with no serious complicetion a7 tops ver 2.1. Continued on P ©, 2020 - 02400 p.m. ~ 04,00 ten pam 100 pam, Paye 7 MEeReaToLOGY AND pa AND PARASITOLOGY ser a! iY 4 b Tt Us an attenuated variola virus * Sietuacy Larne tgs MEBIOLIE FentaLance La a growing eoncmen dn th eatatance to nafctiling methietdiin and wee oxecalt inn PrObTeGtMEMRSAY Le the pore commonly Anewn exemple of thie ‘teoetment of a7. What te the know intloigele EOTEE saaRNtry mechanten for Ue deetonmant ot the + Staph Cassette chromosome mae (CC B. t-lkceanane promiceian TS ") + Plasmid eadLated roplatance ©, Alteration {nthe hacterbal call walt aa " Tolloving the hospital -acqui ted HIN Is the ewerye required mathielt tie a aistinguisiing wraperty of the © compared to the honpieal seem te ‘Ar ieee tratsmteiuia, ene B. Slower tn multiplication C. More tinsue invasiveness 0. More remtatant to antibiotics w of the Jotant mtaph auras (CACHRSA) «What is mann thy -acquired resterant strain 49. Nat La an rHpowrattY vient vieutenge factor 4m the CA A. Enterotoxin Tacros' tn SA HRER ‘B. Pantena-Valontin leveocidin C. Exfoltative toxin ‘ DL eongulase Situation 9 - Interstitial myocarditia is the HOST common carious complication of a protozon} diswase, 50, which of the following 4 this protozoal dinease? A. Toxoplasmoa ls C. Chagas disanne B. Anoebiasis D. Letstimaniasts {9 the etinloyic ayent of this disease? + Leshmania : Trypangsame crud Toxoplasma yondi trypanosome beucel gambionet 52. In what form ara thoy found in the heart muscle? A. Pptmant iqote ¢, Trypomantigote BL Amastigote Ih, Promastigote gituation 10 - A S0-yeare old male patient conphainod of clawing deformity of toch hands which waa confirmed with phyoical examination; both handsuroloyic axaninatian consisted of reduced gonsation of Chetpulp of the small Cinger nnd the mediot aida of the F409 ‘and apparent slarged and aenailive winger ne: Cinger and apparantly eutarde! sioune Macular raehad were also fpearomedial anpectu af batt albown. Ponte mihe volar alia of both forearma. The akin Lesions \ ra SResthatic when pricked with « neadle. he MOST PROWADLE diagnonis? $3. wnat IS C. llexped simplex ; Hansen's Disease . Bt herpes roster D, Guillien-Berre syndrome jo distinguishes the rore elinic-laborstory featur see tyea trom the more benign type of Ene At 54. whic A 3ineidious onest Ae Sjperficial nerve tnvolvenent TQS Ver 2d Continued on Page & = 04100 poms Page @ U, 2020 - 07100 Pp iceos| OLOGY AND van or ers C. wagatiy » Lepeombiy Leak Ds Anesthatic akin len SS whlch of the folteving Yaboratory procedures Lew Ay Serato Bi Earyne tm C1 Polyme b. Skin bloney 4 to estoblish the Y Tenction el Skeometon 11

You might also like